chapter 12 the heart

Pataasin ang iyong marka sa homework at exams ngayon gamit ang Quizwiz!

Dilated cardiomyopathy

moste common cardiomyopathy idiopathic or familial others: ABCCCD chronic Alcohol abuse wet Beriberi Coxsackie B viral myocarditis chronic Cocaine use Chagas disease Doxorubicin toxicity hemochromatosis sarcoidosis peripartum cardiomyopathy HF S3 systolic regurgitant murmur dilated heart on echocardiogram balloon appearance of heart on CXR Na+ restriction ACEI b-blockers diuretics digoxin ICD (implantable cardioverter defibrillator) heart tranplant Takotsubo cardiomyopathy: "broken heart syndrome"—ventricular apical ballooning likely due to increased sympathetic stimulation (stressful situations).

Left heart failure

orthopnea shorteness of breath when supine: ^ venous return from redistribution of blood (immediate gravity effect) exacerbates pulmonary vascular congestion paroxysmal nocturnal dyspnea breathless awakening from sleeping: ^ venous return from redistribution of blood, reabsorption of peripheral edama, etc. pulmonary edema ^ pulmonary venous pressure---pulmponary venous distention and transudation of fluid presence of hemosiderin-laden macrophages (HF cells) in lung

Sinus Bradycardia

1. Rhythm: Regular. 2. Rate: <60 bpm. 3. P wave upright & uniform, precedes each QRS complex. Parasympathetic - vagal dominance of the sinus node 1. Sinus node ischemia - as in inferior wall MI. 2. Oversensitive vagal response - CSM (cervical spondylotic myelopathy), vagal manoeuvre, vomiting, suctioning, severe pain. 3. Increased intracranial pressure. 4. Hypothyroidism 5. Drugs - Beta blockers, calcium antagonists, digitalis, morphine 6. Hypothermia. 1. Atropine 2. Temporary cardiac pacing 3. As alternatives to temporary cardiac pacing: dopamine, epinephrine. 4. Permanent pacing if ongoing symptomatic bradycardia.

Hypertrophic cardiomyopathy

60-70% of cases are familial, autosomal dominant most commonly due to mutations in genes encoding sarcomeric proteins, such as myosin bingding protein C and b-myosin heavy chain can be associated with Friedreich ataxia cause synocope during exercise and may lead to sudden death in young athletes due to ventricular arrhythmia Findings: S4, systolic murmu, may see mitral regurgitation due to impaired mitral valve closure diastolic dysfunction ensues Marked ventricular concentric hypertrophy ( sarcomeres added in paralle) often septal predominance. myofibrillar disarray and fibrosis hypertrophic obstructive cardiomyophathy (subset)---asymmetric septal hypertrophy and systolic anterior motion of mitral valve---outflow obstruction ---dyspnea, possible syncope Treatment: cessation of high intensity athletics, use of b-blocker or non-dihydropyridine Ca2+ channel blockers(eg, verapami). ICD if patient is high risk,

Myxomas

90% occur in the atria "ball valve" obstruction in the left atrium ( associated with multiple syncopal epidodes) early diastolic "tumorplop" sound Histology: gelatinous material, myxoma cells immersed in glycosaminoglycans

A 73-year-old man has noted a change in bowel habits for the past year. Defecation is more difficult and the caliber of stools has decreased. On physical exam, there are no abnormal findings except for stool positive for occult blood. Colonoscopy is performed for the first time in this man, followed by colonic resection with the gross appearance shown in the figure. Which of the following molecular abnormalities has most likely led to these findings? A Acquired APC gene mutation B Homozygous loss of PTEN gene C Inactivation of the RB1 protein by HPV-16 D Mutation in a DNA mismatch repair gene E Tyrosine kinase activation with KIT mutation

A The figure shows an encircling mass that is typical of adenocarcinoma of the descending colon. Such cancers likely to obstruct, but they can also bleed a small amount over months to years, causing iron deficiency anemia. The APC gene, a negative regulator of β-catenin in the WNT signaling pathway, is associated with familial adenomatous polyposis syndrome and most sporadic colon cancers, as in this case. This pathway also is known as the adenoma-carcinoma sequence because the carcinomas develop through an identifiable series of molecular and morphologic steps. Loss of the PTEN tumor suppressor gene is seen in endometrial carcinomas not associated with colon carcinoma and with some hamartomatous polyps of the colon. Evidence for an additional cancer, such as an endometrial cancer, would suggest an inherited mutation in one of the DNA mismatch repair genes, such as MSH2 and MLH1. Homozygous loss of these genes can give rise to right-sided colon cancer and endometrial cancer. Such a mutation is typically associated with microsatellite instability. Infection with some strains of human papillomavirus leads to RB1 protein inactivation and development of cervical carcinoma. Mutation with activation of KIT tyrosine kinase activity occurs in gastrointestinal stromal tumors, which respond well to treatment with imatinib mesylate, a tyrosine kinase inhibitor also used to treat chronic myelogenous leukemia.

A 72-year-old man with poorly controlled diabetes mellitus has worsening exercise tolerance for 5 years. For the past year he has had chest pain with minimal exertion. On physical examination he has bilateral pulmonary rales and pitting edema of his legs. He has an irregular heart rate. A chest radiograph shows prominent right and left heart borders. Echocardiography shows decreased left ventricular ejection fraction (25%) with diminished wall motion. Laboratory studies show an elevated serum B-type natriuretic peptide. Which of the following pathologic findings is most likely present in this man? A Critical coronary stenosis B Left atrial mural thrombus C Hypertrophic cardiomyopathy D Mitral and tricuspid valve thickening E Pericardial fibrinohemorrhagic exudate

A The history of diabetes mellitus and the chest pain put ischemic heart disease at the top of the differential diagnosis list for this man, who has findings with both right and left ventricular failure and enlargement, suggesting ischemic cardiomyopathy. An elevated serum B-type natriuretic peptide (which is measured instead of atrial natriuretic peptide) is consistent with heart failure. Occlusive coronary atherosclerosis may lead to multiple infarctions, or may silently cause progressive myofiber loss, but the end stage is ischemic cardiomyopathy. Atrial mural thrombus formation can occur with aortic valve dysfunction and with dysrrhythmias. Myofiber disarray is characteristic for hypertrophic cardiomyopathy, which affects the interventricular septum preferentially and is usually symptomatic by young adulthood. Although rheumatic valvulitis with thickening may involve both the left and right sides of the heart, this is unusual, and it is not associated with coronary artery disease. Pericardial fluid collection may constrict heart motion, without an enlarged heart, and hemopericardium may acutely occur with ventricular rupture.

A 10-year-old girl develops subcutaneous nodules over the skin of her arms and torso 3 weeks after a bout of acute pharyngitis. She manifests choreiform movements and begins to complain of pain in her knees and hips, particularly with movement. A friction rub is heard on auscultation of her chest. An abnormality detected by which of the following serum laboratory findings is most characteristic of the disease affecting this girl? A Antistreptolysin O antibody titer B Antinuclear antibody titer C Creatinine level D Rapid plasma reagin test E Troponin I level

A Acute rheumatic fever can involve any or all layers of the heart. Because rheumatic fever follows group A streptococcal infections, the antihyaluronidase, anti-DNase, and anti-streptolysin O (ASO) titers are often elevated. The strains of group A streptococci that lead to acute rheumatic fever are less likely to cause glomerulonephritis, so an elevated creatinine level is unlikely. The ANA level could be elevated in systemic lupus erythematosus, which is most likely to produce a serous pericarditis. A positive rapid plasma reagin test suggests syphilis, but the clinical features here are not those of syphilis, and cardiovascular syphilis is one form of tertiary syphilis that develops decades after initial infection. Cardiac troponins are markers for ischemic myocardial injury. Although their levels may be elevated because of the acute myocarditis that occurs in rheumatic fever, this change is not a characteristic of rheumatic heart disease.

A 25-year-old man suffers a sudden cardiac arrest. He is resuscitated. On examination his vital signs are normal. Echocardiography shows that the left ventricle is normal but there is marked thinning with dilation of the right ventricle. MR imaging of his chest shows extensive fibrofatty replacement of the myocardium, but no inflammation. Which of the following is the most likely cause for his findings? A Cardiomyopathy B Chagas disease C Hypertension D Long QT syndrome E Radiation therapy

A Arrhythmogenic right ventricular cardiomyopathy (arrhythmogenic right ventricular dysplasia) is most likely an autosomal dominant inherited condition with abnormal desmosomal adhesion proteins in myocytes. Infections of the heart are accompanied by inflammation, though a late finding in Chagas disease is ventricular fibrosis with ventricular wall thinning. Hypertension leads to ventricular hypertrophy. There is no characteristic gross or microscopic finding with long QT syndrome caused by myocyte channelopathies. Prior radiation therapy results in fibrosis, but it is not likely to be localized to the right ventricle; improving techniques that focus the beam and synchronize it with breathing motion reduce cardiac damage when treating chest cancers.

A 33-year-old woman from Victoria, British Columbia, goes to the physician because of increasingly severe dyspnea, orthopnea, and swelling of the legs for the past 2 weeks. She has no previous history of serious illness or surgery. On physical examination, her temperature is 37.8° C, pulse is 83/min, respirations are 20/min, and blood pressure is 100/60 mm Hg. An ECG shows episodes of ventricular tachycardia. An echocardiogram shows right and left ventricular dilation, but no valvular deformities. An endomyocardial biopsy shows focal myocyte necrosis and lymphocytic infiltrate. Which of the following organisms most likely caused the infection? A Coxsackievirus A B Mycobacterium kansasii C Viridans streptococci D Staphylococcus aureus E Toxoplasma gondii F Trypanosoma cruzi

A Focal myocardial necrosis with a lymphocytic infiltrate is consistent with viral myocarditis. This is uncommon, and many cases may be asymptomatic. In North America, most cases are caused by coxsackieviruses A and B. This illness may often be self-limited. Less often, it ends in sudden death or progresses to chronic heart failure. Mycobacterial infections of the heart are uncommon, but pericardial involvement is the most likely pattern. Septicemia with bacterial infections may involve the heart, but the patient probably would be very ill with multiple organ failure. Viridans streptococci and Staphylococcus aureus are better known as causes of endocarditis with neutrophilic inflammatory infiltrates. Toxoplasma gondii may cause myocarditis with mixed inflammatory cell infiltrates in immunocompromised patients. Trypanosoma cruzi is the causative agent of Chagas disease, seen most often in children. This is probably the most common infectious cause of myocarditis worldwide.

In a clinical study of tetralogy of Fallot, patients are examined before surgery to determine predictors observed on echocardiography that correlate with the severity of the disease and the need for more careful monitoring. A subset of patients is found to have more severe congestive heart failure, poor exercise tolerance, and decreased arterial oxygen saturation levels. Which of the following is most likely to predict a worse clinical presentation for these patients? A Degree of pulmonary stenosis B Diameter of the tricuspid valve C Presence of an atrial septal defect D Size of the ventricular septal defect E Thickness of the left ventricle

A In tetralogy of Fallot, the severity of the obstruction to the right ventricular outflow determines the direction of flow. If the pulmonic stenosis is mild, the abnormality resembles a ventricular septal defect, and the shunt may be from left to right with no cyanosis. With significant pulmonary outflow obstruction, the right ventricular pressure may reach or exceed systemic vascular resistance, and the blood would be shunted from right to left, producing cyanotic heart disease. Even if pulmonic stenosis is mild at birth, the pulmonary orifice does not expand proportionately as the heart grows, and cyanotic heart disease supervenes.

A 60-year-old man has had angina on exertion for the past 6 years. A coronary angiogram performed 2 years ago showed 75% stenosis of the left circumflex coronary artery and 50% stenosis of the right coronary artery. For the past 3 weeks, the frequency and severity of his anginal attacks have increased, and pain sometimes occurs even when he is lying in bed. On physical examination, his blood pressure is 110/80 mm Hg, and pulse is 85/min with irregular beats. An ECG shows ST segment elevation. Laboratory studies show serum glucose, 188 mg/dL; creatinine, 1.2 mg/dL; and troponin I, 1.5 ng/mL. Which of the following is most likely to explain these findings? A Atheromatous plaque fissure with thrombosis B Constrictive pericarditis with calcification C Endomyocardial fibrosis D Extensive myocardial fiber hypertrophy E Left ventricular mural thrombosis F Mitral valve prolapse with regurgitation

A Marked coronary artery occlusion with this degree of stenosis prevents adequate perfusion of the heart when myocardial demand is increased during exertion. He has angina on exertion and recently developed unstable angina, which is manifested by increased frequency and severity of the attacks and angina at rest. The ST segment elevation suggests a developing acute coronary syndrome with myocardial ischemia, but the lack of cardiac enzyme elevation suggests infarction has not yet occurred. In most patients, unstable angina is induced by disruption of an atherosclerotic plaque followed by a mural thrombus and possibly distal embolization, vasospasm, or both. An acute myocardial infarction (MI) can lead to focal fibrinous pericarditis, but it is unlikely to lead to extensive scarring that surrounds the heart. Fibrosis is a late finding from healing of infarction. Hypertrophy of the heart is unlikely to progress significantly in this case because there is neither hypertension nor a valvular lesion. Mural thrombosis may develop on the endocardial surface overlying an infarction, and may fill a ventricular aneurysm following an MI. An acute MI may be complicated by papillary muscle rupture with mitral valve insufficiency.

A 77-year-old woman has had episodes of syncope with exertion for the past month. On physical examination, she is afebrile. Her pulse is 66/min, respirations are 14/min, and blood pressure is 125/85 mm Hg. On auscultation, a systolic ejection murmur is heard. There are a few crackles over the lung bases posteriorly. From the representative gross appearance of the opened aorta shown in the figure, which of the following most likely contributed to the development of this lesion? A Aging B Atherosclerosis C Chromosomal aneuploidy D Hypercalcemia of malignancy E Systemic lupus erythematosus F Tertiary syphilis

A Note that this valve has three cusps, but the nodular deposits of calcium interfere with cusp movement to cause calcific aortic stenosis. This is a degenerative change that may occur in a normal aortic valve with aging. Syncope may occur upon exertion because the stenotic valve prevents stroke volume from increasing in the presence of systemic vasodilation, resulting in hypotension. Atherosclerosis does not produce valvular disease from involvement of the valve itself. Congenital anomalies with chromosomal aneuploidies (e.g., trisomy 21) are unlikely to be associated with aortic stenosis or a bicuspid valve. Hypercalcemia may cause metastatic calcification, but it is unlikely in cardiac valves; it is more likely to cause arrhythmias. Systemic lupus erythematosus may give rise to small sterile vegetations on mitral or tricuspid valves, but these rarely cause valve disease. In syphilis, the aortic root dilates, and aortic insufficiency results.

A 26-year-old woman has had a fever for 5 days. On physical examination, her temperature is 38.2° C, pulse is 100/min, respirations are 19/min, and blood pressure is 90/60 mm Hg. A cardiac murmur is heard on auscultation. Her sensorium is clouded, but there are no focal neurologic deficits. Laboratory findings include hemoglobin, 13.1 g/dL; platelet count, 233,300/mm3; and WBC count, 19,200/mm3. Blood cultures are positive for gram-positive bacteria. Urinalysis shows hematuria. An echocardiogram shows a 1.5-cm vegetation on the mitral valve. Which of the following conditions is this patient most likely to develop? A Cerebral arterial mycotic aneurysm B Dilated cardiomyopathy C Myxomatous mitral valve degeneration D Pericardial effusion with tamponade E Pulmonary abscess

A She developed bacterial septicemia followed by infective endocarditis of the mitral valve. Thus she has a high risk for developing complications of infective endocarditis. Such valvular vegetations are destructive of the valve. The impaired functioning of the mitral valve (most likely regurgitation) would give rise to left atrial dilation and left ventricular failure with pulmonary edema. Septic emboli from the mitral valve vegetation could reach the systemic circulation and give rise to abscesses. Infection of an arterial wall can weaken the wall, resulting in aneurysm formation and the potential for rupture. Dilated cardiomyopathy may be due to chronic alcoholism, or it may be idiopathic. It may be familial, or it may follow myocarditis, but it is not a direct complication of infective endocarditis. Myxomatous degeneration of the mitral valve results from a defect in connective tissue, whether well defined or unknown; the mitral valve leaflets are enlarged, hooded, and redundant. Lesions on the right side of the valve can produce septic emboli that involve the lungs, but vegetations on the left side embolize to the systemic circulation, producing lesions in the spleen, kidneys, or brain. Pulmonary abscesses can occur from right-sided infective endocarditis, because septic emboli pour into the pulmonary arterial circulation.

A 56-year-old man has worsening cough and orthopnea for the past 2 years. On physical examination, he has dullness to percussion at both lung bases and diffuse crackles in the upper lung fields. He is afebrile. Echocardiography shows marked left ventricular hypertrophy and severe aortic stenosis. The representative gross appearance of the opened heart is shown in the figure. A coronary angiogram shows no significant coronary arterial narrowing. Which of the following underlying conditions best accounts for his findings? A Congenital anomaly B Diabetes mellitus C Infective endocarditis D Marfan syndrome E Systemic hypertension

A The bicuspid valve shown has a tendency to calcify with aging, which eventually can result in stenosis, left ventricular hypertrophy, and left-sided heart failure with pulmonary edema. In individuals with congenitally bicuspid valves, symptoms often appear by 50 to 60 years of age. By contrast, calcific aortic stenosis of tricuspid valves manifests in the seventh or eighth decade. Ischemic heart disease, expected with diabetes mellitus, does not lead to valvular stenosis. In infective endocarditis, the patient would have an infection, and the valve would tend to be destroyed, leading to insufficiency. In Marfan syndrome, loss of elastic tissue in the media leads to aortic root dilation, producing aortic valvular insufficiency. Systemic hypertension accounts for left ventricular hypertrophy, but the aortic valve is not affected.

A 31-year-old man experienced chest pain, became increasingly dyspneic and nauseated, and lost consciousness multiple times. Seven days after the appearance of these symptoms, he was found dead in his sleep. External examination of the body by the medical examiner shows no evidence of trauma. The body is 166 cm (5 ft 5 in) in height and weighs 75 kg (BMI 27). The gross appearance of the chest cavity at autopsy with the pericardial sac opened is shown in the figure. What is the most likely underlying cause of his death? A Coronary atherosclerosis B Dilated cardiomyopathy C Disseminated tuberculosis D Systemic sclerosis E Malignant melanoma F Marfan syndrome G Takayasu arteritis

A The figure shows dark red blood filling the opened pericardial cavity, a massive hemopericardium with pericardial tamponade. After excluding trauma, a complication of ischemic heart disease should be suspected. Rupture of a transmural myocardial infarction typically occurs 3 to 7 days after onset, when there is maximal necrosis before significant healing of the infarct. Ischemic heart disease occurs in patients of his age, and risk factors such as obesity, smoking, diabetes mellitus, and hyperlipidemia can play a role in its development. Cardiomyopathies lead to ventricular hypertrophy or dilation, or both, but do not cause rupture. Tuberculosis can cause hemorrhagic pericarditis, typically without tamponade. Scleroderma is most likely to produce serous effusion. Metastases from melanoma and other carcinomas can produce hemorrhagic pericarditis without tamponade. This patient does not have a marfanoid habitus, although Marfan syndrome can cause cystic medial necrosis involving the aorta, leading to aortic dissection that can cause an acute hemopericardium. Takayasu arteritis can involve coronary arteries with aneurysms and rupture, but is most often a rare pediatric condition.

A study of persons receiving emergent medical services is conducted. It is observed that 5% of persons with sudden cardiac arrest who receive cardiopulmonary resuscitation survive. Which of the following is the most likely mechanism for cardiac arrest in these survivors? A Arrhythmia B Infarction C Inflammation D Valve failure E Ventricular rupture

A The most common cause for sudden cardiac arrest is ischemic heart disease. The risk for sudden death is increased with worsening atherosclerotic coronary arterial narrowing. However, the first event with an acute coronary syndrome is typically an arrhythmia, and this is why resuscitation, including defibrillation, can be successful, and survivors may have no ECG or enzyme changes to suggest myocardial infarction has occurred. Inflammation with infarction or infection takes days to develop. A sudden valvular incompetence from papillary muscle rupture, or wall rupture, may complicate an infarction 3 to 7 days following the initial event.

A 53-year-old woman has increasing abdominal girth for the past 2 years. On physical examination she has abdominal distension. An abdominal CT scan shows multiple nodules on peritoneal surfaces along with low attenuation mucinous ascites. Paracentesis is performed and cytologic examination of the fluid obtained shows well-differentiated columnar cells with minimal nuclear atypia. Where did this proliferative process most likely arise in this woman? A Appendix B Jejunum C Ileum D Pancreas E Stomach

A Pseudomyxoma peritonei (PP) is described here. It may arise from low-grade mucinous adenocarcinoma of the appendix, which may be so differentiated that it resembles an appendiceal mucocele. However, PP tends to recur. In women, PP needs to be distinguished from mucinous tumors of the ovary. Mucinous tumors may also arise in the pancreas, but are less likely to disseminate through the peritoneal cavity. Malignancies arising in the small intestine are rare. Mucinproducing malignancies of the stomach are most likely to have a signet ring cell pattern and diffusely infiltrate the gastric wall.

A 19-year-old man is advised to see his physician because genetic screening has detected a disease in other family members. On physical examination, a stool sample is positive for occult blood. A colonoscopy is performed, followed by a colectomy. The figure shows the gross appearance of the mucosal surface of the colectomy specimen. Microscopic examination shows these lesions are tubular adenomas. Molecular analysis of this patient's normal fibroblasts is most likely to show a mutation in which of the following genes? A APC B MLH1 C KRAS D NOD2 E p53

A This young patient's colon shows hundreds of polyps. This is most likely a case of familial adenomatous polyposis (FAP) syndrome, which results from inheritance of one mutant copy of the APC tumor-suppressor gene (a few FAP cases are associated with DNA mismatch repair genes). Every somatic cell of this patient most likely has one defective copy of the APC gene. Polyps are formed when the second copy of the APC gene is lost in many colon epithelial cells. Without treatment, colon cancers arise in 100% of these patients because of accumulation of additional mutations in one or more polyps, typically before 30 years of age. Patients with a gene for hereditary nonpolyposis colorectal carcinoma, such as MLH1 and MSH2, also have an inherited susceptibility to develop colon cancer, but in contrast to patients with FAP, they do not develop numerous polyps. Sporadic colon cancers may have CpG island hypermethylation along with KRAS mutations, whereas others have p53 mutations, but the somatic cells of patients with these cancers do not show abnormalities of these genes. NOD2 mutations are linked with Crohn disease.

Familial adenomatous polyposis

Autosomal dominant mutation of APC tumor suppressor gene on chromosome 5q. 2-hit hypothesis. Thousands of polyps arise starting after puberty; pancolonic; always involves rectum. Prophylactic colectomy or else 100% progress to CRC.

Appendicitis

Acute inflammation of the appendix (yellow arrows in A ), can be due to obstruction by fecalith (red arrow in A ) (in adults) or lymphoid hyperplasia (in children). Initial diffuse periumbilical pain migrates to McBurney point (1 ⁄3 the distance from right anterior superior iliac spine to umbilicus). Nausea, fever; may perforate peritonitis; may elicit psoas, obturator, and Rovsing signs, guarding and rebound tenderness on exam. Differential: diverticulitis (elderly), ectopic pregnancy (use β-hCG to rule out). Treatment: appendectomy.

Spontaneous bacterial peritonitis

Also known as 1° bacterial peritonitis. patients with cirrhosis and ascites Often asymptomatic, but can cause fevers, chills, abdominal pain, ileus, or worsening encephalopathy. Commonly caused by aerobic gram ⊝ organisms, especially E coli. Diagnosis: Paracentesis with ascitic fluid absolute neutrophil count (ANC) > 250 cells/mm3.

Transposition of great vessels

Aorta leaves RV (anterior) and pulmonary trunk leaves LV (posterior)---separation of systemic and pulmonary circulations not compatible with life unless a shunt is present to allow mixing of blood (eg, VSD, PDA or patent foramen ovale) due to failure of the aorticopulmonary septum to spiral without surgical intervention, most infants die within the first few months of life.

A 50-year-old man has had increasing abdominal discomfort and swelling of his legs for the past 2 years. He has smoked cigarettes for 35 years. On physical examination, he has jugular venous distention, even when sitting up. The liver is enlarged and tender and can be palpated 10 cm below the right costal margin. Pitting edema is observed on the lower extremities. A chest radiograph shows bilateral diaphragmatic flattening, pleural effusions, and increased lucency of lung fields. Thoracentesis on the right side yields 500 mL of clear fluid with few cells. Which of the following is most likely to be the underlying disease in this patient? A Acute myocardial infarction B Chronic bronchitis C Primary pulmonary hypertension D Pulmonary valve stenosis E Tricuspid valve stenosis

B The findings point to pure right-sided congestive heart failure. Rarely, this can be caused by right-sided cardiac valvular lesions, such as tricuspid or pulmonic stenosis. Pulmonary hypertension resulting from obstructive lung diseases, such as emphysema or chronic bronchitis, most often caused from smoking cigarettes, is much more common. Primary pulmonary hypertension also can cause right-sided heart failure, but it is a much less common cause than obstructive lung diseases. Because acute myocardial infarction usually affects the left ventricle, left-sided heart failure would be more common in these patients. Chronic left-sided heart failure eventually can lead to right-sided heart failure.

A 35-year-old woman has had palpitations, fatigue, and worsening chest pain during the past year. On physical examination, she is afebrile. Her pulse is 75/min, respirations are 15/min, and blood pressure is 110/70 mm Hg. Auscultation of the chest indicates a midsystolic click with late systolic murmur. A review of systems indicates that the patient has one or two anxiety attacks per month. An echocardiogram is most likely to show which of the following? A Aortic valvular vegetations B Mitral valve prolapse C Patent ductus arteriosus D Pulmonic stenosis E Tricuspid valve regurgitation

B A floppy (prolapsed) mitral valve is usually asymptomatic. When symptomatic, it can cause fatigue, chest pain, and arrhythmias. Some cases are linked to clinical depression and anxiety, and others are associated with Marfan syndrome. Valvular vegetations suggest endocarditis, and a murmur is likely to be heard with infective endocarditis causing valvular insufficiency. A patent ductus arteriosus causes a shrill systolic murmur. Pulmonic stenosis is most often a congenital heart disease. Tricuspid regurgitation is accompanied by a rumbling systolic murmur.

A 3-year-old child is developing normally. Physical examination reveals a low-pitched cardiac murmur. An echocardiogram shows the presence of an ostium secundum, with a 1-cm defect. Which of the following abnormalities is most likely to be found in this child? A Cyanosis at rest B Left-to-right shunt C Mural thrombosis D Pericardial effusion E Pulmonary hypertension

B A persistent ostium secundum is the most common form of atrial septal defect. Because atrial pressures are low, the amount of shunting from the left atrium to the right atrium is small, and this lesion may remain asymptomatic for many years. Eventually, pulmonary hypertension can occur, with reversal of the shunt. Cyanosis is a feature of a right-to-left shunt. A dilated heart with enlarged atria predisposes to mural thrombosis and embolism. Pericardial effusions may occur much later, if congestive heart failure develops.

A 56-year-old man experiences episodes of severe substernal chest pain every time he performs a task that requires moderate exercise. The episodes have become more frequent and severe over the past year, but they can be relieved by sublingual nitroglycerin. On physical examination, he is afebrile, his pulse is 78/min and regular, and there are no murmurs or gallops. Laboratory studies show creatinine, 1.1 mg/dL; glucose, 130 mg/dL; and total serum cholesterol, 223 mg/dL. Which of the following cardiac lesions is most likely to be present in this man? A Calcific aortic stenosis B Coronary atherosclerosis C Restrictive cardiomyopathy D Rheumatic mitral stenosis E Serous pericarditis F Viral myocarditis

B Angina pectoris typically occurs when coronary artery narrowing exceeds 75%. His risk factors include hyperglycemia (diabetes mellitus) and hypercholesterolemia. Calcific aortic stenosis leads to left-sided congestive heart failure (CHF), and the extra workload of the left ventricle may cause angina pectoris. Calcific aortic stenosis (in the absence of a congenital bicuspid aortic valve) is rarely symptomatic at 50 years of age, however. Cardiomyopathies result in heart failure, but without chest pain. Patients with rheumatic heart disease are affected by slowly worsening CHF. Pericarditis can produce chest pain, although not in relation to exercise, and it is not relieved by nitroglycerin. Viral myocarditis may last for weeks, but not for 1 year, and pain may be present at rest.

A 50-year-old man with diabetes mellitus and hypertension has had pain in the left shoulder and arm for the past 12 hours. Over the next 6 hours, he develops shortness of breath, which persists for 2 days. On day 3, he visits the physician. On physical examination, his temperature is 37.1° C, pulse is 82/min, respirations are 18/min, and blood pressure is 160/100 mm Hg. Laboratory studies show total creatine kinase (CK) activity within reference range, but the troponin I level is elevated. He continues to experience dyspnea for the next 3 days. On day 7 after the onset of shoulder pain, he has a cardiac arrest and is resuscitated. Cardiac imaging now shows a large fluid collection around the heart. Which of the following complications has he most likely developed? A Aortic valvular perforation B Hemopericardium C Left ventricular aneurysm D Papillary muscle rupture E Pericarditis

B CK activity begins to increase 2 to 4 hours after an MI, peaks at about 24 to 48 hours, and returns to normal within 72 hours. Troponin I levels begin to increase at about the same time as CK and CK-MB, but remain elevated for 7 to 10 days. Total CK activity is a sensitive marker for myocardial injury in the first 24 to 48 hours. CK-MB offers more specificity, but not more sensitivity. The risk for myocardial rupture is greatest from 4 to 7 days after transmural myocardial necrosis. This patient had an MI on the day of the shoulder pain. When he saw the physician on day 3, the CK levels had returned to normal, but troponin I levels remained elevated. Three days later, the infarct ruptured, and blood filled the pericardial cavity. Cardiac valves are essentially avascular and not subject to ischemic injury. Ventricular aneurysm formation is a late complication of a healed MI. Papillary muscles are at risk for rupture, just like the free wall, but the consequence would be acute valvular insufficiency, not hemopericardium. A transmural MI may lead to pericarditis, often with some accompanying pericardial effusion, but the acute event here in the time frame described suggests rupture.

A 10-year-old girl who is normally developed has chronic progressive exercise intolerance. On physical examination, temperature is 37.1° C, pulse is 70/min, respirations are 14/min, and blood pressure is 100/60 mm Hg. A chest radiograph shows cardiomegaly and mild pulmonary edema. An echocardiogram shows severe left ventricular hypertrophy and a prominent interventricular septum. The right ventricle is slightly thickened. During systole, the anterior leaflet of the mitral valve moves into the outflow tract of the left ventricle. The ejection fraction is abnormally high, and the ventricular volume and cardiac output are both low. Which of the following is the most likely cause of the cardiac abnormalities in this patient? A Autoimmunity against myocardial fibers B β-Myosin heavy chain gene mutation C Deposition of amyloid fibrils D Excessive iron accumulation E Latent enterovirus infection

B Hypertrophic cardiomyopathy is familial in >70% of cases and is usually transmitted as an autosomal dominant trait. The mutations affect genes that encode proteins of cardiac contractile elements. The most common mutation in the inherited forms affects the β-myosin heavy chain. Autoimmune conditions are unlikely to involve the myocardium. Amyloidosis causes restrictive cardiomyopathy. Hemochromatosis can give rise to cardiomyopathy, but it occurs much later in life. Viral infections produce generalized inflammation and cardiac dilation.

A 17-year-old girl jumps up for a block in the third match of a volleyball tournament and suddenly collapses. She requires cardiopulmonary resuscitation. A similar episode occurs a month later. She had been healthy all her life and complained only of limited episodes of chest pain in games during the current school year. Which of the following pathologic findings of the heart is most likely to be present in this girl? A Extensive myocardial hemosiderin deposition B Haphazardly arranged hypertrophied septal myocytes C Large, friable vegetations with destruction of aortic valve cusps D Mitral valvular stenosis with left atrial enlargement E Tachyzoites within foci of myocardial necrosis and inflammation

B Hypertrophic cardiomyopathy is the most common cause of sudden unexplained death in young athletes. There is asymmetric septal hypertrophy that reduces the ejection fraction of the left ventricle, particularly during exercise. Histologically, haphazardly arranged hypertrophic myocardial fibers are seen. Arrhythmias can occur. If persons with this condition survive to adulthood, chronic heart failure may develop. Hemochromatosis gives rise to a cardiomyopathy in middle age. Valve destruction with vegetations is seen in infective endocarditis and would be accompanied by signs of sepsis. Rheumatic heart disease with chronic valvular changes would be unusual in a patient this age, and the course is most often slowly progressive. Tachyzoites of Toxoplasma gondii signify myocarditis, a process that may occur in immunocompromised individuals.

A 15-year-old boy complains of pain in his legs when he runs more than 300 m. Physical examination shows temperature, 36.8° C; pulse, 76/min; respirations, 22/min; and blood pressure, 165/90 mm Hg. The radial pulses are 4+, and the dorsalis pedis pulses are 1+. Arterial blood gas measurement shows a normal oxygen saturation level. Which of the following congenital cardiovascular anomalies is most likely to be present in this patient? A Aortic valve stenosis B Coarctation of the aorta C Patent ductus arteriosus D Transposition of the great arteries E Tricuspid valve atresia

B In children and adults, the coarctation is typically postductal, and collateral branches from the proximal aorta supply the lower extremities, leading to the large pulse differential between upper and lower extremities. Collaterals often involve intercostal arteries whose enlargement produces "rib notching" on chest radiographs. Diminished renal blood flow below the coarctation increases renin production and promotes hypertension. Aortic valve stenosis causes left-sided heart failure and no pressure differential in the extremities. A patent ductus arteriosus produces a small left-to-right shunt. Transposition results in a right-to-left shunt with cyanosis. Tricuspid valve atresia affects the right side of the heart.

An 86-year-old man has had increasing dyspnea and reduced exercise tolerance for the past 7 years. On physical examination, he is afebrile and has a blood pressure of 135/85 mm Hg. An irregularly irregular heart rate averaging 76/min is audible on auscultation of the chest. Crackles are heard at the bases of the lungs. A chest radiograph shows mild cardiomegaly and mild pulmonary edema. Echocardiography shows slight right and left ventricular wall thickening with reduced left and right ventricular wall motion, reduced left ventricular filling, and an ejection fraction estimated to be 25%. An endomyocardial biopsy specimen shows amorphous pink-staining deposits between myocardial fibers, but no inflammation and no necrosis. Echocardiography would most likely show which of the following functional cardiac disturbances? A Dynamic obstruction to ventricular outflow B Impaired ventricular diastolic filling C Increased end-systolic volume D Mitral and tricuspid valvular insufficiency E Reduced ejection fraction

B Reduced cardiac chamber compliance is a feature of the restrictive form of cardiomyopathy. Cardiac amyloidosis may be limited to the heart (so-called senile cardiac amyloidosis derived from transthyretin protein) or may be part of organ involvement in systemic amyloidosis derived from serum amyloid-associated (SAA) protein or, in multiple myeloma, derived from light chains (AL amyloid). Incidental isolated atrial deposits of amyloid are derived from atrial natriuretic peptide. Myocardial fiber dysfunction markedly reduces ventricular compliance. Dynamic left ventricular outflow obstruction is characteristic of hypertrophic cardiomyopathy. Valvular insufficiency of mitral and tricuspid valves can occur with dilated cardiomyopathy, which also reduces contractility and ejection fraction with increased end-systolic volume.

A 20-year-old woman in her ninth month of pregnancy has increasing pain on defecation and notices bright red blood on the toilet paper. She has had no previous gastrointestinal problems. After she gives birth, the rectal pain subsides, and there is no more bleeding. Which of the following is the most likely cause of these findings? A Angiodysplasia B Hemorrhoids C Intussusception D Ischemic colitis E Volvulus

B Hemorrhoids are a common problem that can stem from any condition that increases venous pressure and causes dilation of internal or external hemorrhoidal veins above and below the anorectal junction. Angiodysplasia of the colon leads to intermittent hemorrhage, typically in older individuals. Ischemic colitis is rare in young individuals because the most common underlying cause (severe atherosclerotic disease involving mesenteric vessels) occurs in older patients. Intussusception and volvulus are rare causesof mechanical bowel obstruction; they occur suddenly in adults and are surgical emergencies.

A 33-year-old man has a routine health maintenance examination. A stool sample is positive for occult blood. On colonoscopy, a 6-cm ulcerative lesion is seen projecting into the cecum. There are three smaller sessile lesions from 1 to 3 cm in size. The microscopic appearance of a section of the ulcerated lesion is shown in the figure. The smaller lesions are reported as sessile serrated adenomas. Which of the following molecular biological events is thought to be most critical in the development of such lesions? A Amplification of ERBB2 gene B Defective DNA mismatch repair gene C Germline transmission of a defective RB1 gene D Overexpression of E-cadherin gene E Translocation of retinoic acid receptor alpha gene

B The lesion is an adenocarcinoma, showing irregular glands infiltrating the muscle layer. Such a lesion in a 30-yearold man strongly indicates a hereditary predisposition. One hereditary form of cancer is called hereditary nonpolyposis colorectal cancer (HNPCC) and results from defective DNA mismatch repair genes. As a result, mutations accumulate in microsatellite repeats (microsatellite instability) that lead to loss of transforming growth factor beta (TGF-β) receptormediated control of colonic epithelial cell proliferation and loss of proapoptotic BAX protein enhancing survival of these transformed cells. He could have taken NSAIDs that inhibit COX-2 expressed in most colonic adenomas and carcinomas. In contrast to familial adenomatous polyposis syndrome, HNPCC does not lead to the development of hundreds of polyps in the colon. Detection of ERBB2 (HER2/NEU) expression is important in breast cancers. Germline inheritance of the tumor suppressor gene RB1 predisposes to retinoblastoma and osteosarcoma, not colon carcinoma. E-cadherin is required for intercellular adhesion; its levels are reduced, not increased, in carcinoma cells. Translocation of the retinoic acid receptor alpha gene is characteristic of acute promyelocytic leukemia.

bacterial endocarditis

Bacteria from jane fever roth spots osler nodes murmur Janeway lesions Anemia nail-bed hemorrhage emboli Acute---S aureus high virulence large vegetations on previously normal valves Subacute---viridans streptococci low virulence smaller vegetations on congenitally abnormal or diseased valves sequela of dental procedures S bovis (gallolyticus) colon cancer S epidermidis on prosthetic valves Endocarditis may also be nonbacterial (marantic/thrombotic) 2° to malignancy, hypercoagulable state, or lupus Mitral valve is most frequently involved. Tricuspid valve endocarditis is associated with IV drug abuse (don't "tri" drugs). Associated with S aureus, Pseudomonas, and Candida. Culture ⊝; most likely Coxiella burnetii, Bartonella spp., HACEK (Haemophilus, Aggregatibacter (formerly Actinobacillus), Cardiobacterium, Eikenella, Kingella)

A 73-year-old woman has had episodes of chest pain during the past week. She is afebrile. Her pulse is 80/min, respirations are 16/min, and blood pressure is 110/70 mm Hg. On auscultation of the chest, heart sounds seem distant, but the lung fields are clear. Neck veins are distended to the angle of the jaw, even while sitting. There is a darkly pigmented, irregular, 1.2-cm skin lesion on the right shoulder. A chest radiograph shows prominent borders on the left and right sides of the heart. Pericardiocentesis yields bloody fluid. Laboratory findings include a serum troponin I level of 0.3 ng/mL. Which of the following lesions is the most likely cause of these findings? A Calcific aortic stenosis B Coronary atherosclerosis C Epicardial metastases D Mitral valvulitis E Tuberculous pericarditis

C Hemorrhagic pericardial effusion most commonly is caused by either tumor or tuberculosis. The most common neoplasm involving the heart is metastatic cancer, because primary cardiac neoplasms are rare. The most common primary sites are nearby—lung, breast, and esophagus. The skin lesion in this patient is likely to be a malignant melanoma, which tends to metastasize widely, including to the heart. Most cardiac metastases involve the epicardium/pericardium. (By convention, even though epicardial surfaces are often involved most severely, the term pericardial effusion is typically used when fluid is present, or pericarditis is used when inflammation is present.) A large effusion can cause tamponade, which interferes with cardiac motion. Calcific aortic stenosis leads to left-sided congestive heart failure, with pulmonary edema as a key finding. Coronary atherosclerosis may lead to myocardial infarction, which can be complicated by ventricular rupture and hemopericardium, but the level of troponin I in this case suggests that infarction did not occur. Rheumatic heart disease mainly affects the cardiac valves, but acute rheumatic fever can produce fibrinous pericarditis. Tuberculosis is unlikely in this case because no pulmonary lesions were seen on the radiograph.

tetralogy of Fallot

Caused by anterosuperior displacement of the infundibular septum. Most common cause of early childhood cyanosis PROVe Pulmonary infundibular stenosis (most important determinant for prognosis) Right ventricular hypertrophy boot shape heart Overriding aorta VSD Squatting: ^ SVR, ! right to left shunt, improves cyanosis tel spells: crying, fever, and exercise due to exacerbation of RV outflow obstruction

A 48-year-old, previously healthy woman reports having suddenly lost consciousness four times in the past 6 months. In three instances, she was unconsciousness for only a few minutes. After the fourth episode 1 month ago, she was unconscious for 6 hours and had weakness in her right arm and difficulty speaking. On physical examination, she is afebrile, and her blood pressure is normal. No murmurs are auscultated. She has good carotid pulses with no bruits. Which of the following cardiac lesions is most likely to be present in this woman? A Bicuspid aortic valve B Coronary artery thrombosis C Left atrial myxoma D Mitral valve stenosis E Pericardial effusion

C Atrial myxoma is the most common primary cardiac neoplasm. On the left side of the heart, it can produce a ball-valve effect that intermittently occludes the mitral valve, leading to syncopal episodes and possible strokes from embolization to cerebral arteries. Calcification of a bicuspid valve can lead to stenosis and heart failure, but this condition is progressive. Coronary artery thrombosis results in an acute ischemic event, typically with chest pain. By the time left atrial enlargement with mural thrombosis and risk of embolization occurs from mitral stenosis, this patient would have been symptomatic for years. Most pericardial effusions are not large and do not cause major problems. Large effusions could lead to tamponade, but this is not an intermittent problem.

A 44-year-old, previously healthy man has experienced worsening exercise tolerance accompanied by marked shortness of breath for the past 6 months. On physical examination, his vital signs are normal. He has diffuse rales in all lung fields and pitting edema to the knees. Laboratory studies show serum sodium, 130 mmol/L; potassium, 4 mmol/L; chloride, 102 mmol/L; CO2, 25 mmol/L; creatinine, 2 mg/dL; and glucose, 120 mg/dL. A 100-mL urine sample is collected. There is 1.3 mmol sodium and 40 mg creatinine in the urine sample. A chest radiograph shows cardiomegaly and pulmonary edema with pleural effusions. An echocardiogram shows four-chamber cardiac enlargement and mitral and tricuspid valvular regurgitation, with an ejection fraction of 30%. A coronary angiogram shows less than 10% narrowing of the major coronary arteries. Which of the following is the most likely diagnosis? A Amyloidosis B Hypercholesterolemia C Familial cardiomyopathy D Rheumatic heart disease E Trypanosoma cruzi infection

C Congestive heart failure with four-chamber dilation is suggestive of dilated cardiomyopathy; implicated in causation are genetic factors (in 20% to 50% of cases), myocarditis, and alcohol abuse. The patient's fractional excretion of sodium is less than 1%, consistent with prerenal azotemia Many cases of dilated cardiomyopathy have no known cause. Dilation is more prominent than hypertrophy, although both are present, and all chambers are involved. Amyloidosis produces restrictive cardiomyopathy. Hypercholesterolemia would predispose to atherogenesis with coronary artery narrowing and ischemic heart disease. Rheumatic heart disease would most often produce some degree of valvular stenosis, often with some regurgitation, and the course usually is more prolonged. Chagas disease from T. cruzi infection affects the right ventricle more often than the left.

A 68-year-old man has become increasingly lethargicand weak for the past 7 months. On physical examination, his temperature is 36.9° C, pulse is 70/min, respirations are 15/min, and blood pressure is 160/105 mm Hg. On auscultation of his chest, a friction rub is audible. There are no other remarkable findings. The representative gross appearance of the heart is shown in the figure. Which of the following laboratory findings is most likely to be reported for this patient? A Elevated serum anti-streptolysin O titer B Elevated plasma renin level C Increased blood urea nitrogen level D Increased serum CK-MB level E Positive ANA with "rim" pattern F Positive viral serology

C Fibrinous pericarditis leads to the rough, corrugated brownish surfaces of epicardium and reflected pericardial sac as shown, which is sometimes described as a "bread and butter" appearance (after dropping the buttered bread on the carpet). Friction between epicardial and pericardial surfaces yields the rub, which may disappear with fluid collection (serofibrinous pericarditis). The most common cause is uremia resulting from renal failure. Elevation of the anti-streptolysin O titer accompanies rheumatic fever. Acute rheumatic fever may produce fibrinous pericarditis, but rheumatic fever is uncommon at this age. An elevated renin level is seen in some forms of hypertension, but by itself does not indicate renal failure. Elevation of serum creatine kinase occurs in myocardial infarction. An acute myocardial infarction may be accompanied by a fibrinous exudate over the area of infarction, not the diffuse pericarditis seen in this patient. A positive ANA test result suggests a collagen vascular disease, such as systemic lupus erythematosus, more likely associated with a serous pericarditis (without extensive fibrinous exudate). Fibrinous pericarditis is unlikely the result of an infection, but a fibrinopurulent appearance could suggest bacterial infection.

A 62-year-old woman has had increasing dyspnea for the past 2 years. She now awakens at night with air hunger and cough productive of frothy sputum. On examination, she has rales in all lung fields. Her point of maximal impulse is strong and displaced laterally. Echocardiography shows a decreased ejection fraction of 30% with concentric increase in left ventricular wall size. The valves appear normal. Which of the underlying diseases does she have? A Amyloidosis B Cardiomyopathy C Hypertension D Myocarditis E Pericarditis

C Left-sided congestive heart failure leads to pulmonary congestion and edema. Systemic hypertension is common and leads to pressure load with predominantly concentric left ventricular hypertrophy with systolic dysfunction. The other listed options are more likely to produce diastolic dysfunction and decrease of both right and left ventricular function.

A 45-year-old man receives a cardiac allograft for dilated cardiomyopathy. He has no problems with rejection, but 5 years later he has worsening exercise tolerance with increasing dyspnea and peripheral edema. Echocardiography shows a reduced ejection fraction of 35%. Which of the following pathologic abnormalities has he most likely developed in the allograft? A Amyloidosis B Constrictive pericarditis C Coronary arteriopathy D Non-Hodgkin lymphoma E Toxoplasmosis

C Nearly every allograft develops some degree of arteriopathy within 10 years, and half of patients have significant arteriopathy by 5 years following transplantation. Unlike atherosclerosis, the smaller coronary artery branches are preferentially affected, but the result is the same: ischemic damage. The inflammation that may come from rejection is not a risk for amyloid deposition. A transplanted heart does not have a functional pericardial sac, and though there may be some fibrous adhesions following surgery, they are not constrictive. Transplant recipients receiving immunosuppressive therapy have immune dysregulation that increases the risk for both carcinomas and lymphoid malignancies, but these are unlikely to involve the heart. The immunosuppression with antirejection drugs increases the risk for infection with opportunistic agents, but these are not common and they can often be treated.

A 45-year-old man experiences crushing substernal chest pain after arriving at work one morning. Over the next 4 hours, the pain persists and begins to radiate to his left arm. He becomes diaphoretic and short of breath, but waits until the end of his 8-hour shift to go to the hospital. An elevated serum value of which of the following laboratory tests would be most useful for diagnosis of this patient on admission to the hospital? A ALT B AST C CK-MB fraction D C-reactive protein E LDH-1 F Lipase

C Of the enzymes listed, CK-MB is the most specific for myocardial injury from the acute myocardial infarction (MI) described in this patient. The levels of this enzyme begin to increase within 2 to 4 hours of ischemic myocardial injury. ALT elevation is more specific for liver injury. AST is found in various tissues; elevated levels are not specific for myocardial injury. The elevation of lactate dehydrogenase (LDH)-1 compared with LDH-2 suggests myocardial injury, but LDH activity peaks 3 days after an MI. C-reactive protein is elevated with inflammatory processes, but is nonspecific; it has been used as a predictor of acute coronary syndromes. Lipase is a marker for pancreatitis.

A 37-year-old woman has the sudden onset of chest pain. On examination she is afebrile but tachycardic and hypotensive. An ECG shows ST segment elevation and pathologic Q waves. The representative microscopic appearance of her left circumflex artery is shown in the figure. Which of the following underlying conditions is she most likely to have? A Acute myelogenous leukemia B Chronic alcoholism C Diabetes mellitus D Marfan syndrome E Polyarteritis nodosa

C The figure shows a coronary artery with marked luminal narrowing caused by atheromatous plaque, complicated by a recent thrombus filling the narrowed lumen. Atherosclerosis is accelerated with diabetes mellitus. When a premenopausal woman develops severe atherosclerosis, as in this case, underlying diabetes mellitus or a lipid disorder must be strongly suspected. Patients with leukemias may have reversal of any atheromas, but can develop hypercoagulable states. When this occurs, there is widespread thrombosis in normal blood vessels. Individuals with chronic alcoholism often have less atherosclerosis than individuals of the same age who do not consume large amounts of alcohol. The cystic medial necrosis that occurs in Marfan syndrome most often involves the ascending aorta and predisposes to dissection that could involve coronary arteries, although with external compression. Polyarteritis nodosa can involve coronary arteries and give rise to coronary thrombosis when the arterial wall is necrotic and inflamed.

A 48-year-old woman has had increasing dyspnea for the past 2 days. She experiences sudden cardiac arrest. The representative light microscopic appearance of her left ventricular free wall is shown in the figure. Which of the following is the most likely diagnosis? A Acute rheumatic myocarditis B Cardiomyopathy C Myocardial infarction D Septic embolization E Viral myocarditis

C The figure shows intensely eosinophilic myocardial fibers with loss of nuclei, all are indicative of coagulative necrosis. The deeply red-stained transverse bands are called contraction bands. Neutrophils infiltrate between myocardial fibers. This pattern is most likely caused by a myocardial infarction (MI) that is approximately 24 to 48 hours old. Chest pain is present in most but not all cases of MI. Rheumatic myocarditis is characterized by minimal myocardial necrosis with foci of granulomatous inflammation (Aschoff bodies). There is no significant inflammation with restrictive cardiomyopathies such as amyloidosis or hemochromatosis. Septic emboli result in focal abscess formation. In viral myocarditis, there is minimal focal myocardial necrosis with round cell infiltrates.

A 56-year-old man has experienced increased fatigue and decreased exercise tolerance for the past 2 years. On physical examination, his temperature is 37° C, pulse is 75/min, respirations are 17/min, and blood pressure is 115/75 mm Hg. On auscultation, diffuse crackles are audible. The abdomen is distended with a fluid wave, and there is bilateral pitting edema to the knees. A chest radiograph shows pulmonary edema, pleural effusions, and marked cardiomegaly. An echocardiogram shows mild tricuspid and mitral regurgitation and reduced right and left ventricular wall motion, with an ejection fraction of 30%. He experiences cerebral, renal, and splenic infarctions over the next year. Chronic use of which of the following substances has most likely produced these findings? A Acetaminophen B Cocaine C Ethanol D Lisinopril E Nicotine F Propranolol

C The findings point to dilated cardiomyopathy (DCM) with both right-sided and left-sided heart failure. The most common toxin producing DCM is alcohol, and individuals with chronic alcoholism are more likely to have DCM than to have ischemic heart disease. Acetaminophen ingestion can be associated with hepatic necrosis and analgesic nephropathy. Cocaine can produce ischemic effects on the myocardium. Lisinopril is an angiotensin-converting enzyme inhibitor that is used to treat hypertension. Nicotine in cigarette smoke is a risk factor for atherosclerosis. Propranolol is a β-blocker that has been used to treat hypertension, and it may exacerbate bradycardia and congestive heart failure.

A 19-year-old man suddenly collapses and is brought to the emergency department. His vital signs are temperature, 37.1° C; pulse, 84/min; respirations, 18/min; and blood pressure, 80/40 mm Hg. Laboratory findings include hemoglobin, 13.5 g/dL; platelet count, 252,000/mm3; WBC count, 7230/ mm3; serum glucose, 73 mg/dL; and creatinine, 1.2 mg/dL. The total creatine kinase (CK) level is elevated, with a CK-MB fraction of 10%. Which of the following underlying conditions is most likely to be present in this patient? A DiGeorge syndrome B Down syndrome C Familial hypercholesterolemia D Hereditary hemochromatosis E Marfan syndrome

C The laboratory findings suggest an acute myocardial infarction. Individuals with familial hypercholesterolemia have accelerated and advanced atherosclerosis, even by the second or third decade. DiGeorge syndrome can be associated with various congenital heart defects, but survival with this syndrome is usually limited by infections resulting from cell-mediated immunodeficiency. Down syndrome (trisomy 21) is often accompanied by endocardial cushion defects, not ischemic heart disease. Hereditary hemochromatosis may result in an infiltrative cardiomyopathy with iron overload, more typically by the fifth decade. Marfan syndrome may result in aortic dissection or floppy mitral valve.

A 21-year-old primigravida gives birth at term to a 2800-g infant with no apparent external anomalies. The next day, the infant develops increasing respiratory distress and cyanosis. Echocardiography reveals a slitlike left ventricular chamber, small left atrium, and atretic aortic and mitral valves. Through which of the following structures could blood from the lungs most likely have reached the infant's systemic circulation? A Anomalous venous return B Foramen ovale C Patent ductus arteriosus D Right fourth aortic arch E Truncus arteriosus F Ventricular septal defect

C These findings are compatible with , which may have varying degrees of severity, ranging from severe (as in this case, with virtually no function on the left side of the heart) to milder degrees of hypoplasia. Most of the oxygenated blood returning to the left atrium is shunted across the foramen ovale back to the lungs, increasing pulmonary flow and decreasing oxygenation. Less oxygenated blood exiting the right ventricle into the pulmonic trunk can shunt through a patent ductus arteriosus to the aorta to supply the systemic circulation. Anomalous venous return does not generally connect to the aorta, and there still must be a connection from the lungs to the aorta. The right fourth aortic arch rarely persists. Truncus arteriosus is an anomalous, incomplete separation of the pulmonic and aortic trunks. If there is virtually no left ventricular chamber, a ventricular septal defect would not provide any significant flow.

Molecular pathogenesis of colorectal cancer

Chromosomal instability pathway: mutations in APC cause FAP and most sporadic CRC (via adenoma-carcinoma sequence; (firing) order of events is AK-53). Microsatellite instability pathway: mutations or methylation of mismatch repair genes (eg, MLH1) cause Lynch syndrome and some sporadic CRC (via serrated polyp pathway). Overexpression of COX-2 has been linked to colorectal cancer, NSAIDs may be chemopreventive.

DiGeorge syndrome

Chromosome 22q11 deletion. Aberrant development of 3rd and 4th pouches !T-cell deficiency (thymic aplasia) and hypocalcemia (failure of parathyroid development). Associated with cardiac defects (conotruncal anomalies).

Cardiac tamponade

Compression of the heart by fluid---! CO. Equilibration of diastolic pressure in all 4 chambers Findings: Beck triad: hypotension, distended neck vein, distant heart sound ^ HR pulsus paradoxus ECG: low voltage QRS and electrial alternans (due to "swinging"movement of heart in larg effusion)

A 2-year-old child had an illness 1 year ago characterized by a high fever. Staphylococcus epidermidis was cultured from the blood. The child was given antibiotic therapy and recovered. Now on physical examination, a harsh, waxing and waning, machinery-like murmur is heard on auscultation of the upper chest. A chest radiograph shows prominence of the pulmonary arteries. Echocardiography shows all cardiac valves to be normal in configuration. Laboratory studies show normal arterial oxygen saturation level. Which of the following congenital heart diseases is most likely to explain these findings? A Aortic atresia B Aortic coarctation C Atrial septal defect D Patent ductus arteriosus E Tetralogy of Fallot F Total anomalous pulmonary venous return

D Although often not causing a large shunt defect, a patent ductus arteriosus can produce a significant murmur and predispose to infection. This left-to-right shunt may eventually result in pulmonary hypertension. An atretic valve has no flow across it and does not produce a murmur, but there would be a murmur across a shunt around the atretic valve. Aortic atresia is not compatible with continued survival, as seen in hypoplastic left heart syndrome. Aortic coarctations by themselves produce no shunting and no pulmonary hypertension. An atrial septal defect is unlikely to produce a loud murmur because of the minimal pressure differential between the atria. Because pulmonic stenosis is a component of tetralogy of Fallot, no pulmonary hypertension results, and the right-to-left shunting can lead to cyanosis with decreased arterial oxygen saturation. Total anomalous pulmonary venous return is not accompanied by a murmur because of the low venous pressure.

A 50-year-old man has sudden onset of severe substernal chest pain that radiates to the neck. On physical examination, he is afebrile, but has tachycardia, hyperventilation, and hypotension. No cardiac murmurs are heard on auscultation. Emergent coronary angiography shows a thrombotic occlusion of the left circumflex artery and areas of 50% to 70% narrowing in the proximal circumflex and anterior descending arteries. Which of the following complications of this disease is most likely to occur within 1 hour of these events? A Myocardial rupture B Pericarditis C Valvular insufficiency D Ventricular fibrillation E Thromboembolism

D In the period immediately after coronary thrombosis, arrhythmias are the most important complication and can lead to sudden cardiac death. It is believed that, even before ischemic injury manifests in the heart, there is greatly increased electrical irritability predisposing to dysrhythmias. Myocardial rupture, valvular insufficiency from papillary muscle involvement, and pericarditis occur several days later. Another complication is a left ventricular aneurysm, a late complication of the healing of a large transmural infarction; a mural thrombus may fill an aneurysm and become a source of emboli. If portions of the coronary thrombus break off and embolize, they enter smaller arterial branches in the distribution already affected by ischemia. Valvular insufficiency from a ruptured papillary muscle would occur later in the course

A 41-year-old woman has had increasing dyspnea for the past week. On physical examination, temperature is 37.3° C, pulse is 85/min, respirations are 20/min, and blood pressure is 150/95 mm Hg. There is dullness to percussion over the lung bases. A chest radiograph shows large bilateral pleural effusions and a normal heart size. Laboratory findings include serum creatinine, 3.1 mg/dL; urea nitrogen, 29 mg/dL; troponin I, 0.1 ng/mL; WBC count, 3760/mm3; hemoglobin, 11.7 g/dL; and positive ANA and anti-double-stranded DNA antibody test results. Which of the following cardiac lesions is most likely to be present in this patient? A Calcific aortic stenosis B Hemorrhagic pericarditis C Nonbacterial thrombotic endocarditis D Libman-Sacks endocarditis E Mural thrombosis F Rheumatic verrucous endocarditis

D Libman-Sacks endocarditis is an uncommon complication of systemic lupus erythematosus (SLE) that has minimal clinical significance because the small vegetations, although they spread over valves and endocardium, are unlikely to embolize or cause functional flow problems. Calcific aortic stenosis may be seen in older individuals with tricuspid valves, or it may be a complication of bicuspid valves. Although pericardial effusions are common in active SLE, along with pleural effusions and ascites from serositis, they are usually serous effusions, and no significant hemorrhage or scarring occurs. The vegetations of nonbacterial thrombotic endocarditis are prone to embolize. Mural thrombi are most likely to form when cardiac chambers are dilated, or there is marked endocardial damage. Rheumatic heart disease is an immunologic disease based on molecular mimicry; serologic tests would be positive for anti-streptolysin O (ASO), not ANA.

A 73-year-old woman had an episode a week ago in which she became disoriented, had difficulty speaking, and had persisting weakness on the right side of her body. On physical examination, she is now afebrile with pulse of 68/min, respirations of 15/min, and blood pressure of 130/85 mm Hg. On auscultation, the lungs are clear, the heart rate is irregular, and there is a midsystolic click. A chest CT scan shows a focus of bright attenuation within the heart. An echocardiogram shows that one valvular leaflet appears to balloon upward. The ejection fraction is estimated to be 55%. Laboratory findings show serum creatine kinase (CK), 100 U/L; glucose, 77 mg/dL; creatinine, 0.8 mg/dL; calcium, 8.1 mg/dL; and phosphorus, 3.5 mg/dL. Which of the following is the most likely diagnosis? A Carcinoid heart disease B Hyperparathyroidism C Infective endocarditis D Mitral annular calcification E Rheumatic heart disease F Senile calcific stenosis

D Mitral annular calcification is often an incidental finding on chest radiograph, echocardiography, or at autopsy. Larger accumulations of calcium in the mitral ring can impinge on the conduction system, however, causing arrhythmias or disrupting the endocardium to provide a focus for infective endocarditis and thrombus formation (which can embolize and cause a stroke, as in this patient). Some cases are associated with mitral valve prolapse. Carcinoid heart disease leads to endocardial and valvular collagenous thickening. Hyperparathyroidism can cause metastatic calcification, which usually does not involve the heart, and deposits would not be so focal; this patient does not have hypercalcemia. Infective endocarditis is a destructive process, and healing may lead to fibrosis, but not to nodular calcium deposition. The most common infiltrative cardiomyopathies are hemochromatosis and amyloidosis. Rheumatic heart disease can lead to scarring with some calcium deposition, but the valve leaflets undergo extensive scarring, with shortening and thickening of the chordae tendineae that preclude upward prolapse. Senile calcific stenosis involves the aortic valve; in this case, there is no evidence of stenosis.

A 65-year-old healthy woman has a check of her health status and the only finding is a midsystolic click on auscultation of the heart. Within 5 years she has increasing dyspnea. Echocardiography now shows mitral regurgitation from prolapse of a leaflet. Which of the following pathologic changes is most likely present in this valve? A Destructive vegetations B Dystrophic calcification C Fibrinoid necrosis D Myxomatous degeneration E Rheumatic fibrosis

D Myxomatous mitral valve degeneration can be primary from a connective tissue disorder such as Marfan syndrome or secondary to chronic hemodynamic forces (later in life); the chordae tendineae become elongated and can rupture to produce acute valvular incompetence. Destructive vegetations occur with infective endocarditis, and develop over days to weeks. Dystrophic calcification in older persons can occur in the mitral annulus or aortic valve; the former is typically incidental and the latter may produce symptomatic stenosis. Fibrinoid necrosis is most typical of hyperplastic arteriolosclerosis, not cardiac valves. Rheumatic heart disease leads to valvular scarring with shortening and thickening of the chordae tendineae, not thinning and elongation.

A 19-year-old man has had a low-grade fever for 3 weeks. On physical examination, his temperature is 38.3° C, pulse is 104/min, respirations are 28/min, and blood pressure is 95/60 mm Hg. A tender spleen tip is palpable. There are splinter hemorrhages under the fingernails and tender hemorrhagic nodules on the palms and soles. A heart murmur is heard on auscultation. Which of the following infectious agents is most likely to be cultured from this patient's blood? A Coxsackievirus B B Mycobacterium tuberculosis C Pseudomonas aeruginosa D Viridans streptococci E Trypanosoma cruzi

D Prolonged fever, heart murmur, mild splenomegaly, and splinter hemorrhages suggest a diagnosis of infective endocarditis. The valvular vegetations with infective endocarditis are friable and can break off and embolize. The time course of weeks suggests a subacute form of bacterial endocarditis resulting from infection with a less virulent organism, such as viridans streptococci. Group A streptococci are better known as a cause for rheumatic heart disease, with noninfectious vegetations. Pseudomonas aeruginosa is more likely to cause an acute form of bacterial endocarditis that worsens over days, not weeks; this organism is more common as a nosocomial infection or it may occur in injection drug users. Coxsackievirus B and Trypanosoma cruzi are causes of myocarditis. Tuberculosis involving the heart most often manifests as pericarditis.

A 52-year-old woman has had a chronic cough for the past 2 years, accompanied by a small amount of occasionallyblood-streaked, whitish sputum. On physical examination, her temperature is 37.9° C, pulse is 72/min, respirations are 22/min, and blood pressure is 125/80 mm Hg. Crackles are heard on auscultation over the upper lung fields. Heart sounds are faint, and there is a 15 mm Hg inspiratory decline in systolic arterial pressure. The chest radiograph shows prominent heart borders with a "water bottle" configuration. Pericardiocentesis yields 200 mL of bloody fluid. Infection with which of the following organisms is most likely to produce these findings? A Candida albicans B Coxsackievirus B C Group A streptococcus D Mycobacterium tuberculosis E Staphylococcus aureus

D The clinical features are those of pericarditis with effusion, and the most common causes of hemorrhagic pericarditis are metastatic carcinoma and tuberculosis. An effusion of this size is sufficient to produce some cardiac tamponade that diminishes cardiac output; the paradoxical drop in pressure (more than 10 mm Hg) is called pulsus paradoxus and can be caused by pericarditis and by tamponade. Candida is a rare cardiac infection in immunocompromised individuals. Coxsackieviruses are known to cause myocarditis. Group A streptococci are responsible for rheumatic fever; in the acute form, rheumatic fever can lead to fibrinous pericarditis, and in the chronic form, it can lead to serous effusions from congestive heart failure. Staphylococcus aureus is most often a cause of infective endocarditis.

A 5-year-old child is not as active as other children his age. During the past 8 months, the child has had multiple episodes of respiratory difficulty following exertion. On physical examination, his temperature is 37° C, pulse is 81/min, respirations are 19/min, and blood pressure is 95/60 mm Hg. On auscultation, a loud holosystolic murmur is audible. There are diffuse crackles over the lungs bilaterally, with dullness to percussion at the bases. A chest radiograph shows a prominent left heart border, pulmonary interstitial infiltrates, and blunting of the costodiaphragmatic recesses. The representative gross appearance of the child's heart is shown in the figure. Which of the following additional pathologic conditions would most likely develop in this child? A Aortic regurgitation B Coronary atherosclerosis C Nonbacterial thrombotic endocarditis D Pulmonary hypertension E Restrictive cardiomyopathy

D The figure shows a large ventricular septal defect. By the age of 5 years, such an uncorrected defect causes marked shunting of blood from left to right, causing pulmonary hypertension (Eisenmenger complex). The left and right ventricular chambers undergo hypertrophy and some dilation, but the functioning of the cardiac valves is not greatly affected. In most cases, congenital heart disease is not an antecedent to ischemic heart disease. Nonbacterial thrombotic endocarditis most often occurs secondary to a hypercoagulable state in adults. Restrictive cardiomyopathy may occur from conditions such as amyloidosis or hemochromatosis.

A 27-year-old woman gives birth to a term infant after an uncomplicated pregnancy and delivery. The infant is cyanotic at birth. Two months later, physical examination shows the infant to be at the 37th percentile for height and weight. The representative gross appearance of the infant's heart is shown in the figure. What is the most likely diagnosis? A Aortic stenosis B Pulmonic stenosis C Tetralogy of Fallot D Transposition of the great vessels E Truncus arteriosus

D The figure shows that the aorta emerges from the right ventricle, and the pulmonic trunk exits the left ventricle, consistent with complete transposition of the great vessels. Unless there is another anomalous connection between the pulmonary and systemic circulations, this condition is incompatible with extrauterine life. The most common additional anomalous connections would be ventricular septal defect, patent ductus arteriosus, and patent foramen ovale (or atrial septal defect). In pulmonic and aortic stenosis, the great arteries are normally positioned, but small. In tetralogy of Fallot, the aorta overrides a ventricular septal defect, but is not transposed. In truncus arteriosus, the spiral septum that embryologically separates the great arteries does not develop properly.

An infant born at term is noted to have cyanosis during the first week of life. On examination a heart murmur is auscultated. Abnormal findings with echocardiography include an overriding aorta, ventricular septal defect, right ventricular thickening, and pulmonic stenosis involving the fetal heart. This infant is most likely to have an inherited mutation involving which of the following genes? A β-Myosin heavy chain (β-MHC) B Fibrillin 1 (FBN1) C KCNQ1 D NOTCH2 E Transthyretin (TTR)

D There are a number of mutations in genes linked to congenital heart disease that encode proteins in transcription or signaling pathways. The NOTCH pathway plays a role in modulation of vascular development, including cardiac outflow tracts. NOTCH2 mutations are associated with tetralogy of Fallot, as in this infant. β-MHC gene mutations are associated with some cases of hypertrophic cardiomyopathy. Fibrillin-1 (FBN1) gene mutations underlie Marfan syndrome. KCNQ1 mutations may be seen with long QT syndrome. One form of cardiac amyloidosis is linked to transthyretin (TTR) gene mutations.

An 11-year-old boy had a sore throat, no cough, tonsillar exudates, and 38.3° C fever 3 weeks ago, and a throat culture was positive for group A β-hemolytic Streptococcus. On the follow-up examination, the child is afebrile. His pulse is 85/min, respirations are 18/min, and blood pressure is 90/50 mm Hg. On auscultation, a diastolic mitral murmur is audible, and there are diffuse rales over both lungs. Over the next 2 days he has several episodes of atrial fibrillation accompanied by signs of acute left ventricular failure. Which of the following pathologic changes occurring in this child's heart is most likely to be the cause of the left ventricular failure? A Amyloidosis B Fibrinous pericarditis C Mitral valve fibrosis D Myocarditis E Tamponade F Verrucous endocarditis

D This boy developed acute left ventricular failure, an uncommon but serious complication of acute rheumatic fever. Pancarditis with pericarditis, endocarditis, and myocarditis develop during the acute phase. Myocarditis led to dilation of the ventricle so severe that the mitral valve became incompetent. Rheumatic heart disease is now uncommon, and the number of children that require prophylactic antibiotic therapy to prevent just one case is >10,000. Chronic inflammatory conditions may produce reactive systemic amyloidosis, but this is unlikely to occur given the limited and episodic nature of the streptococcal infection that causes rheumatic heart disease. Fibrinous pericarditis can produce an audible friction rub, but it is not constrictive, and the amount of fluid and fibrin are not great, so no tamponade occurs. Myocardial necrosis associated with myocarditis is patchy, and the ventricle does not rupture to produce tamponade. Fibrosis and fusion of the mitral valve leaflets develop over weeks to months and indicate chronic rheumatic valvulitis. Verrucous vegetations are small and may produce a murmur, but they do not interfere greatly with valve function and do not tend to embolize.

A 77-year-old woman fell and fractured her ankle. She has spent most of her time in bed for the past 16 days. She develops sudden chest pain, dyspnea, and diaphoresis. On examination she has left thigh swelling and tenderness. A chest CT shows areas of decreased attenuation in the right and left pulmonary arteries. A day later she has difficulty speaking. MR angiography shows focal occlusion of a left middle cerebral artery branch. Which of the following cardiac abnormalities is she most likely to have? A Atrial myxoma B Infective endocarditis C Nonbacterial thrombotic endocarditis D Patent foramen ovale E Ventricular aneurysm

D This is the infamous "paradoxical embolus" that has appeared far more often in question sets than in real life. She started with thrombophlebitis that led to pulmonary embolism, but there must be an explanation for the "stroke" that then occurred. Pulmonary emboli can obstruct the pulmonary arterial circulation, raising right atrial pressure, and opening a patent foramen ovale that normally remains closed because of higher left atrial pressure. The remaining choices do not explain pulmonary thromboembolism. A left atrial myxoma can embolize to the brain; lesions of endocarditis are most often on the left side of the heart and could produce cerebral emboli; a ventricular aneurysm is virtually always on the left side of the heart because it results from a healed infarction, and can be filled with mural thrombus that can embolize.

A 44-year-old woman has had increasing abdominal distention for the past 6 weeks. On physical examination, there is an abdominal fluid wave, and bowel sounds are present. Paracentesis yields 1000 mL of slightly cloudy serous fluid. Cytologic examination of the fluid shows malignant cells consistent with adenocarcinoma. Molecular analysis of these cells shows an MSH2 gene mutation with microsatellite instability. Her medical history indicates that she has had no major medical illnesses and no surgical procedures. Her sister was diagnosed with endometrial cancer and her brother had carcinoma of the stomach. Which of the following conditions is the most likely cause of this patient's symptoms? A Angiodysplasia B Crohn disease C Diverticulosis D Lynch syndrome E Peptic ulcer disease

D Of the conditions listed, the one most likely to lead to adenocarcinoma in a patient of this age is hereditary nonpolyposis colorectal cancer, or Lynch syndrome. Crohn disease is unlikely because the patient has not had prior serious illness, and Crohn disease of long duration is unlikely to remain asymptomatic. Although adenocarcinoma may complicate Crohn disease, it does not occur as frequently as in ulcerative colitis. This explains why colectomy is often performed for ulcerative colitis, but bowel resections are avoided, if possible, in Crohn disease. The other conditions listed are not premalignant.

Evolution of myocardial infarction 1-3 days

Gross: hyperemia Light microscope extensive coagulative necrosis acut inflammation with neutrophils contraction band necross complication postinfarction fibrinous pericarditis

Evolution of myocardial infarction 3-14 days

Gross: hyperemic border central yellow-brown softening Light microscope macrophages then granulation tissue at margins complication Free wall rupture ---tamponade papillary muscle rupture---mitral regurgitation; interventricular septal rupture due to macrophage-mediated structural degradation. LV pseudoaneurysm (risk of rupture).

A 44-year-old woman with rheumatic heart disease with aortic stenosis undergoes valve replacement with a bioprosthesis. She remains stable for the next 8 years and then develops diminished exercise tolerance. Which of the following complications involving the bioprosthesis has most likely occurred? A Embolization B Hemolysis C Myocardial infarction D Paravalvular leak E Stenosis

E Bioprostheses made from pig valves are subject to wear and tear. The leaflets may calcify, resulting in stenosis, or they may perforate or tear, leading to insufficiency. Thrombosis with embolization is unlikely to occur with bioprostheses that are indicated for persons who cannot receive anticoagulant therapy; it is an uncommon complication of mechanical prostheses, lessened by anticoagulant therapy. Hemolysis is not seen in bioprostheses and is rare in modern mechanical prostheses. Myocardial infarction from embolization or from a poorly positioned valve is rare. Paravalvular leaks are rare complications of the early postoperative period.

A 5-year-old girl who is below the 5th percentile for height and weight for age has exhibited easily fatigability since infancy. On physical examination, she appears cyanotic. Her temperature is 37° C, pulse is 82/min, respirations are 16/min, and blood pressure is 105/65 mm Hg. Pulse oximetry shows decreased oxygen saturation. One month later, she has fever and obtundation. A cerebral CT scan shows a right parietal, ring-enhancing, 3-cm lesion. Which of the following congenital heart diseases is the most likely diagnosis? A Atrial septal defect B Bicuspid aortic valve C Coarctation of the aorta D Patent ductus arteriosus E Truncus arteriosus F Ventricular septal defect

E Cyanosis at this early age suggests a right-to-left shunt. Truncus arteriosus, transposition of the great arteries, and tetralogy of Fallot are the most common causes of cyanotic congenital heart disease. The cerebral lesion suggests an abscess as a consequence of septic embolization from infective endocarditis, which can complicate congenital heart disease. Atrial septal defect, patent ductus arteriosus, and ventricular septal defect initially lead to left-to-right shunts, though the shunt may reverse with development of pulmonary hypertension. Coarctation is not accompanied by a shunt and cyanosis. In most cases, a bicuspid valve is asymptomatic until adulthood, and there is no shunt.

A 55-year-old man undergoes orthotopic cardiac transplantation. Two months later, an endomyocardial biopsy specimen shows focal myocardial cell death with scattered perivascular lymphocytes and plasma cells. He is treated with sirolimus. Which of the following pathologic processes best accounts for these biopsy findings? A Autoimmunity B Autophagy C Ischemia D Infection E Rejection

E Endomyocardial biopsies are routinely performed after cardiac transplantation to monitor possible immune rejection, and acute cellular rejection amenable to therapy with immunosuppression is described here. Turnover of cellular organelles occurs constantly by autophagy, generating lipofuscin pigment in the cells, but this slow process is not a feature of rejection. The transplant is foreign tissue to the host, so rejection is not an autoimmune process. Months to years later, coronary arteriopathy characteristic of cardiac transplantations may produce ischemic changes. Infection is a definite possibility because of the immunosuppressive drugs administered to control the rejection process, although plasma cells are not a key feature of acute infection.

A 22-year-old previously healthy man undergoes a tooth extraction, and 4 days later he develops a fever. On physical examination his temperature is 37.6° C. A high-pitched systolic murmur is auscultated. A tentative diagnosis of congenital heart disease is made. In which of the following locations is the congenital anomaly in this man most likely found? A Ascending aorta B Atrial appendage C Chordae tendineae D Cusps of valves E Muscular septum

E Infective endocarditis is present in this man. If there is a known cardiovascular congenital anomaly, then dental procedures may be preceded by prophylactic antibiotic therapy. The systolic murmur suggests a left-sided lesion, with left-to right shunt. A small ventricular septal defect (VSD) may not lead to significant shunting of blood and remain subclinical, but it still represents a risk for endocarditis. Only a murmur may provide a clue, and the higher pitch goes with a smaller defect. Most VSDs occur in the membranous septum, but about 10% are in the muscular septum, and this difference may be due to closure of many muscular defects during life.

A retrospective study of myocardial infarction is performed to analyze patterns of cardiac injury. One pattern of injury involves the posterior left ventricular wall and septum. Which of the following pathologic abnormalities is most likely to produce this pattern? A Ascending aortic dissection B Left anterior descending arterial plaque rupture C Left circumflex arterial vasculitis D Right coronary sinus embolization E Right posterior descending arterial thrombosis

E Myocardial infarction results from occlusion of large coronary arterial branches, and in most cases an occluding thrombus is present. The posterior left ventricle and septum are supplied by the posterior descending artery. The left circumflex artery supplies the lateral left ventricular wall, whereas the left anterior descending artery supplies the anterior left ventricle. An aortic dissection that extends proximally may cause tamponade, compressing the heart, great vessels, and even coronary arteries, but this is much less likely a cause for myocardial infarction than atherosclerotic coronary arterial disease. The coronary sinus is where venous blood from the myocardium drains into the right atrium.

A 41-year-old woman has been awakened at night with "air hunger" for the past year. She notes sleeping better while sitting up in bed. Her serum B-type natriuretic peptide is >400 pg/mL (very high). What cardiac disease best explains her condition? A Atrial myxoma B Fibrinous pericarditis C Giant cell myocarditis D Libman-Sacks endocarditis E Rheumatic valvulitis

E Paroxysmal nocturnal dyspnea is a feature of leftsided congestive heart failure, and rheumatic heart disease most often involves the mitral, aortic, or both valves, and left-sided valvular disease leads to pulmonary edema. While upright, pulmonary edema fluid is more concentrated at lung bases, which helps improve breathing. Rheumatic heart disease was more common before antibiotic therapy for group A β-hemolytic streptococcal infections was available, and multiple bouts beginning in childhood led to valvular damage over decades. An atrial myxoma usually occurs on the left side of the heart, but the obstruction is often intermittent. Fibrinous pericarditis can produce chest pain, but the amount of accompanying fluid is often small so that cardiac function is not impaired. Giant cell myocarditis is a rare cause of cardiac failure. Libman-Sacks endocarditis, seen in systemic lupus erythematosus, typically does not impair valvular or ventricular function.

A study of ischemic heart disease analyzes cases of individuals hospitalized with acute chest pain in which myocardial infarction was documented at autopsy. The gross and microscopic appearances of the hearts are correlated with the degree of coronary atherosclerosis and its complications, clinical symptoms, and therapies given before death. Hemorrhage and contraction bands in necrotic myocardial fibers are most likely to be seen with infarction in which of the following settings? A Acute coronary vasculitis B Anti-arrhythmic drug usage C Angioplasty with stent placement D Septic embolization E Thrombolytic therapy

E Reperfusion of an ischemic myocardium by spontaneous or therapeutic thrombolysis changes the morphologic features of the affected area. Reflow of blood into vasculature injured during the period of ischemia leads to mitochondrial dysfunction, followed by leakage of blood into the tissues (hemorrhage). Contraction bands are composed of closely packed hypercontracted sarcomeres. They are most likely produced by exaggerated contraction of previously injured myofibrils that are exposed to a high concentration of calcium ions from the plasma. The damaged cell membrane of the injured myocardial fibers allows calcium to penetrate the cells rapidly. Free radical formation and release of leukocyte enzymes further potentiate myocardial cell death. Hemorrhage would not be a prominent feature in the other listed options. Vasculitides involving the heart are uncommon; Takayasu arteritis can involve coronary arteries, but is most often a rare pediatric condition. Drugs used to control arrhythmias during acute coronary syndromes are unlikely to have hemorrhage as an adverse event. Angioplasty per se does not increase the risk for hemorrhage, and stents help to keep the artery open longer. Septic embolization from infected valvular vegetations to a coronary artery is uncommon, although such emboli may produce focal necrosis and hemorrhage.

A 71-year-old woman has had a 10-kg weight loss accompanied by severe nausea and vomiting of blood for the past 8 months. On physical examination, she is afebrile. Laboratory studies show hemoglobin, 8.4 g/dL; platelet count, 227,100/mm3; and WBC count, 6180/mm3. Biopsy specimens obtained by upper gastrointestinal endoscopy show adenocarcinoma of the stomach. CT scan of the abdomen shows multiple hepatic masses. CT scan of the head shows a cystic area in the right frontal lobe. Her condition is stable until 2 weeks later, when she develops severe dyspnea. A chest CT scan shows areas of decreased pulmonary arterial attenuation. Which of the following cardiac lesions is most likely to be present in this woman? A Calcific aortic valvular stenosis B Constrictive pericarditis C Epicardial metastatic carcinoma D Left ventricular mural thrombosis E Nonbacterial thrombotic endocarditis

E So-called marantic vegetations may occur on any cardiac valve, but tend to be small and do not damage the valves. They have a tendency to embolize, however. They can occur with hypercoagulable states that accompany certain malignancies, especially mucin-secreting adenocarcinomas. Thrombosis can occur anywhere, but is most common in leg veins, predisposing to pulmonary thromboembolism. This paraneoplastic state is known as Trousseau syndrome. Calcific aortic stenosis occurs at a much older age, usually in the eighth or ninth decade, and produces obstruction but not embolism. Cardiac metastases are uncommon, and they tend to involve the epicardium; they do not explain embolism with cerebral infarction in this case. A metastatic tumor can encase the heart to produce constriction, but this is rare. Mural thromboses occur when cardiac blood flow is altered, as occurs in a ventricular aneurysm or dilated atrium, but persons with malignancies likely have no or minimal ischemic heart disease.

A 25-year-old man was found dead at home by the apartment manager, who had been called by the decedent's employer because of failure to report to work for the past 3 days. An external examination by the medical examiner showed splinter hemorrhages under the fingernails and no signs of trauma. The gross appearance of the heart at autopsy is shown in the figure. Which of the following laboratory findings is most likely to provide evidence for the cause of his disease? A Elevated anti-streptolysin O titer B Positive ANCA serology C Increased creatine kinase-MB (CK-MB) fraction D High double-stranded DNA autoantibody titer E Positive blood culture for Staphylococcus aureus

E The aortic valve shown has large, destructive vegetations. The probe passes through a perforated leaflet, typical of infective endocarditis caused by highly virulent organisms such as Staphylococcus aureus. The verrucous vegetations of acute rheumatic fever are small and nondestructive, and the diagnosis is suggested by an elevated anti-streptolysin O titer. A positive ANCA determination suggests a vasculitis, which is unlikely to involve cardiac valves. An elevated creatine kinase-MB level suggests myocardial, not endocardial, injury. A positive double-stranded DNA finding suggests systemic lupus erythematosus, which can produce nondestructive Libman-Sacks endocarditis.

A 59-year-old man has experienced chronic fatigue for the past 18 months. On physical examination, he is afebrile. A chest radiograph shows bilateral pulmonary edema and a prominent left heart border. The representative gross appearance of his heart is shown in the figure. Laboratory studies show serum glucose, 74 mg/dL; total cholesterol, 189 mg/dL; total protein, 7.1 g/dL; albumin, 5.2 g/dL; creatinine, 6.1 mg/dL; and urea nitrogen, 58 mg/dL. What is the most likely diagnosis? A Chronic alcoholism B Diabetes mellitus C Hemochromatosis D Pneumoconiosis E Systemic hypertension

E The markedly thickened left ventricular wall is characteristic of myocardial fiber hypertrophy caused by increased pressure load from hypertension, which often is associated with chronic renal disease. Left ventricular failure leads to pulmonary edema. Chronic alcoholism is most often associated with dilated cardiomyopathy. Diabetes mellitus accelerates atherosclerosis, leading to ischemic heart disease and myocardial infarction; the normal glucose level does not fit with diabetes mellitus. Hemochromatosis leads to dilated cardiomyopathy. Pneumoconioses produce restrictive lung disease with cor pulmonale and predominantly right ventricular hypertrophy.

A 14-year-old girl has fever and chest pain 2 weeks after having a mild upper respiratory tract infection. On physical examination, her temperature is 37° C, pulse is 90/min, respirations are 20/min, and blood pressure is 85/45 mm Hg. A friction rub is audible on auscultation of the chest. A chest radiograph shows pulmonary edema. An echocardiogram shows small vegetations at the closure line of the mitral and aortic valves. An endomyocardial biopsy shows focal interstitial aggregates of mononuclear cells enclosing areas of fibrinoid necrosis. Her condition improves over the next month. The representative gross appearance of the affected heart is shown in the figure. Which of the following cardiac abnormalities is most likely to occur in this patient? A Constrictive pericarditis B Dilated cardiomyopathy C Left ventricular aneurysm D Myxoma E Valvular stenosis

E The mitral valve in the figure shows shortening and thickening of the chordae tendineae typical of chronic rheumatic valvulitis, and the small verrucous vegetations (arrowheads) are characteristic of superimposed acute rheumatic fever. Valvular scarring can follow years after initial group A streptococcal infection. Rheumatic heart disease develops after the immune response directed against the bacterial antigens (similar to cardiac antigens, and thus a form of molecular mimicry) damages the heart because streptococcal antigens cross-react with the heart. The mitral and aortic valves are most commonly affected, so right ventricular dilation from tricuspid involvement is less likely. In almost all cases, the fibrinous pericarditis seen during the acute phase with friction rub resolves without significant scarring, and constrictive pericarditis does not typically develop. Although there is myocarditis with acute rheumatic fever, it does not lead to dilated cardiomyopathy. A left ventricular aneurysm is a complication of ischemic heart disease. Primary cardiac neoplasms, including myxoma, are rare and not related to infection.

A 66-year-old man has had cough and worsening shortness of breath for 3 years. On examination, there is dullness to percussion at both lung bases and poorly audible breath sounds. On physical examination, pulse is 77/min and BP is 110/80 mm Hg. He does not have anginal pain. His liver span is increased to 14 cm. He has pitting edema to his knees. Jugular venous distention is noted to the angle of the jaw at 45-degree elevation of his head while lying down. Which of the following is most likely causing his heart disease? A Atrial myxoma B Essential hypertension C Hyperlipidemia D Rheumatic fever E Smoking

E These findings are consistent with right-sided congestive heart failure leading to peripheral edema, body cavity effusions (pleural effusions in this case), passive congestion of the liver, and jugular venous distension. Pure right-sided failure is less common than left-sided failure, and the former most often follows pulmonary disease (cor pulmonale). Chronic obstructive pulmonary disease (COPD) is more common than restrictive lung disease, and smoking leads to COPD. Atrial myxomas are uncommon, more often on the left side of the heart, and may produce intermittent valvular obstruction. Essential hypertension is systemic and places a pressure load on the left side of the heart. Hyperlipidemia is a risk factor for ischemic heart disease that is more likely to involve the left side of the heart. Rheumatic heart disease may produce heart failure, but it is more often left-sided.

Following an uncomplicated pregnancy, a term infant appears normal at birth, but at 1 day of life the infant develops respiratory distress. On physical examination the infant has tachypnea, tachycardia, and cyanosis. There is an S1 ejection click and a split S2 with prominent P sound. A radiograph shows normal heart size but prominent hilar vascular markings. Echocardiography shows a small left atrium, large right atrium, normally sized ventricles, widely patent foramen ovale, and normally positioned aorta and pulmonary trunk. What type of congenital heart disease does this infant most likely have? A Atrial septal defect B Coarctation of the aorta, preductal type C Patent ductus arteriosus D Tetralogy of Fallot E Total anomalous pulmonary venous connection

E This shunt results from abnormal confluence of pulmonary veins leading to the right atrium (or systemic veins), and not the left. Obstruction is often present, as in this case, with pulmonary congestion. Deoxygenated systemic and oxygenated pulmonary venous blood mix in the right atrium. There must be an atrial septal defect for blood to reach the left atrium, but an ASD by itself does not explain this case. A coarctation is not associated with cyanosis; a preductal coarctation is life-threatening. Patent ductus arteriosus is a left-to-right shunt without cyanosis. Tetralogy of Fallot can lead to cyanosis, but mixing of blood occurs at an overriding aorta.

A 68-year-old woman has had increasing dyspnea and orthopnea for the past year. She does not report any chest pain. On physical examination, her temperature is 37° C, pulse is 77/min, respirations are 20/min, and blood pressure is 140/90 mm Hg. On auscultation of the chest, diffuse crackles are heard in all lung fields. No murmurs or gallops are heard, and the heart rate is regular. A chest radiograph shows prominent right and left heart borders. Coronary angiography shows 90% occlusion of the left anterior descending artery. Echocardiography shows no valvular abnormalities, but there is decreased left ventricular wall motion and an ejection fraction of 32%. Laboratory studies show serum glucose of 81 mg/ dL, creatinine of 1.6 mg/dL, total cholesterol of 280 mg/dL, triglyceride of 169 mg/dL, and troponin I of 1 ng/mL. Which of the following pharmacologic agents is most likely to be beneficial in the treatment of this patient? A Amiodarone B Alteplase C Glyburide D Nitroglycerin E Propranolol F Simvastatin

F An ischemic cardiomyopathy can result from coronary atherosclerosis, but she does not have an acute coronary syndrome. The major identifiable risk factor in this case is hypercholesterolemia, and the HMG-CoA reductase inhibitors (the statin drugs) are helpful to lower cholesterol, specifically LDL cholesterol. Amiodarone is used to treat intractable arrhythmias. Glyburide is used in the treatment of type 2 diabetes mellitus, but this patient is not hyperglycemic. Nitroglycerin is a vasodilator used to treat angina. Propranolol is a β-blocker that has been used to treat hypertension, and it may exacerbate bradycardia and congestive heart failure. Altreplase (tissue plasminogen activator) is used early in treatment of coronary thrombosis to help reestablish coronary blood flow.

A neonate developing normally has a newborn checkup. On physical examination, there is a systolic murmur. Echocardiography reveals a muscular defect of the intraventricular septum. A checkup 30 years later fails to reveal either a murmur or a flow defect between the ventricles. Which of the following cells most likely proliferated and led to disappearance of the defect? A Adipocytes B Conduction cells C Endothelial cells D Fibroblasts E Mesothelial cells F Stem cells

F Native cardiac stem cells can proliferate and replace cardiac myocytes throughout life, but these stem cells are most active in neonates. They have the potential to respond to injury. Some ventricular septal defects do close during life. Research is ongoing regarding methods for inducing stem cell proliferation. adipocytes enlarge with aging. Endothelial cells may proliferate to produce more coronary collateral channels in response to exercise training, and they can resurface vascular grafts. Fibroblasts respond to injury by producing collagenous scar tissue that reduces contractility.

A 69-year-old man with metabolic syndrome had chest pain and an elevated serum troponin I level 1 year ago. He was treated in the hospital with anti-arrhythmic agents for 1 week. An echocardiogram showed an ejection fraction of 28%. He now has markedly reduced exercise tolerance. On physical examination, his temperature is 37° C, pulse is 68/min, respirations are 17/min, and blood pressure is 130/80 mm Hg. Diffuse crackles are heard on auscultation of the lungs. The representative gross appearance of his heart is shown in the figure. Which of the following complications of this disease is the patient most likely to develop? A Atrial myxoma B Cardiac tamponade C Constrictive pericarditis D Hypertrophic cardiomyopathy E Infective endocarditis F Systemic thromboembolism

F The figure shows an enlarged and dilated heart with a large ventricular aneurysm with a thin wall and white fibrous endocardial surface. Such an aneurysm most likely results from weakening of the ventricular wall at the site of a prior healed myocardial infarction. Because of the damage to the endocardial lining, with stasis and turbulence of blood flow in the region of the aneurysm, mural thrombi are likely to develop. When detached, thrombi in the left side of the heart embolize to the systemic circulation and can cause infarcts elsewhere. An atrial myxoma is the most common primary cardiac neoplasm, but it is rare and is not related to ischemic heart disease. Cardiac rupture with tamponade is most likely to occur 5 to 7 days after an acute myocardial infarction. Constrictive pericarditis follows a previous suppurative or tuberculous pericarditis. Hypertrophic cardiomyopathy is not related to ischemic heart disease, but 50% of cases are familial and may be related to genetic mutations in genes encoding for cardiac contractile elements. Infective endocarditis is more likely to complicate valvular heart disease or septal defects.

An 82-year-old woman has had increasing fatigue for the past 2 years. During this time, she has experienced paroxysmal dizziness and syncope. On physical examination, she is afebrile. Her pulse is 44/min, respirations are 16/min, and blood pressure is 100/65 mm Hg. On auscultation, the lungs are clear, and no murmurs are heard. An echocardiogram shows a normal-sized heart with normal valve motion and estimated ejection fraction of 50%. After parasympathetic (vagal) stimulation, the heart rate slows and becomes irregular. An abnormality involving which of the following is most likely to be present in this patient? A Atrioventricular node B Bundle of His C Left bundle branch D Parasympathetic ganglion E Right bundle branch F Sinoatrial node G Sympathetic ganglion

F The pacemaker for the heart is the sinoatrial (SA) node, with a natural rhythm near 70/min and a normal range of 60/min to 100/min. Other parts of the cardiac conduction system pass along this rate. Rates less than 60/min are defined as bradycardia, and rates greater than 100/min are defined as tachycardia. Bradyarrhythmias less than 50/min suggest an SA node disorder. SA node dysfunction may worsen with cardioactive drugs, such as cardiac glycosides, β-adrenergic blockers, calcium channel blockers, and amiodarone. An increase in sinus rate results from an increase in sympathetic tone acting via β-adrenergic receptors or a decrease in parasympathetic tone acting via muscarinic receptors, or both. Abnormalities involving the other listed options are unlikely to produce such a pronounced and consistent bradycardia.

A 20-year-old woman has had nausea and vague lower abdominal pain for the past 24 hours, but now the pain has become more severe. On physical examination, the pain is worse in the right lower quadrant, and there is rebound tenderness. A stool sample is negative for occult blood. Abdominal plain film radiographs show no free air. The result of a serum pregnancy test is negative. Which of the following laboratory findings is most useful to aid in the diagnosis of this patient? A Entamoeba histolytica cysts in the stool B Hyperamylasemia C Hypernatremia D Increased serum alkaline phosphatase E Increased serum carcinoembryonic antigen F Neutrophilia with left shift

F Acute appendicitis can be accompanied by an elevated WBC count with neutrophilia and left shift. This is helpful but not decisive, and the decision to operate must be based on clinical judgment. Amebiasis is most likely associated with a history of diarrhea, often with blood in the stool. Hyperamylasemia occurs in acute pancreatitis. Diarrhea with fluid loss and dehydration can lead to hypernatremia. The serum carcinoembryonic antigen level may be increased in patients with colonic cancers; however, this test is not specific for colon cancer. The alkaline phosphatase level may be increased in biliary tract obstruction.

A 59-year-old man with a lengthy history of chronic alcoholism has noticed increasing abdominal girth for the past 6 months. He has had increasing abdominal pain for the past 2 days. On physical examination, his temperature is 38.2° C. Examination of the abdomen shows a fluid wave and prominent caput medusae over the skin of the abdomen. There is diffuse abdominal tenderness. An abdominal plain film radiograph shows no free air. Paracentesis yields 500 mL of cloudy yellow fluid. Gram stain of the fluid shows gram-negative rods. Which of the following is the most likely diagnosis? A Appendicitis B Collagenous colitis C Diverticulitis D Ischemic colitis E Pseudomembranous colitis F Spontaneous bacterial peritonitis

F Spontaneous bacterial peritonitis is an uncommon complication found in about 10% of adult patients with cirrhosis of the liver and ascites. The ascitic fluid provides an excellent culture medium for bacteria, which can invade the bowel wall or spread hematogenously to the serosa. Spontaneous bacterial peritonitis also can occur in children, particularly children with nephrotic syndrome and ascites. The most common organism cultured is Escherichia coli. Appendicitis has a peak incidence in younger patients; the pain is often (but not always) more localized in the right lower quadrant, and ascites is usually absent. Appendicitis is not related to alcoholism. Collagenous colitis is uncommon; it most often leads to watery diarrhea in middle-aged women. Diverticulitis with rupture could produce peritonitis, but there is typically no ascites, and diverticulitis is not related to alcoholism. Ischemic colitis may produce infarction with rupture and peritonitis, but ascites is usually lacking, and individuals with chronic alcoholism are unlikely to have marked atherosclerosis. Pseudomembranous colitis is a complication of antibiotic therapy.

Pectinate (dentate) line

Formed where endoderm (hindgut) meets ectoderm. Above pectinate line—internal hemorrhoids, adenocarcinoma. Arterial supply from superior rectal artery (branch of IMA). Venous drainage: superior rectal vein inferior mesenteric vein splenic vein portal vein. Internal hemorrhoids receive visceral innervation and are therefore not painful. Lymphatic drainage to internal iliac lymph nodes. Below pectinate line—external hemorrhoids, anal fissures, squamous cell carcinoma. Arterial supply from inferior rectal artery (branch of internal pudendal artery). Venous drainage: inferior rectal vein internal pudendal vein internal iliac vein common iliac vein IVC. External hemorrhoids receive somatic innervation (inferior rectal branch of pudendal nerve) and are therefore painful if thrombosed. Lymphatic drainage to superficial inguinal nodes. Anal fissure—tear in the anal mucosa below the Pectinate line. Pain while Pooping; blood on toilet Paper. Located Posteriorly because this area is Poorly Perfused. Associated with lowfiber diets and constipation.

Evolution of myocardial infarction 2 weeks- several months

Gross: Gray-white Light microscope contracted scar complete complication Dressler syndrome HF arrhythmias true ventricular aneurysm (risk of mural thrombus).

Evolution of myocardial infarction 0-24h

Gross: dark mottling pale with tetrazolium stain ( normal is blue) Light microscope early coagulative necrosis edema hemorrhage wave fibers Neutrophils appear reperfusion injury contraction band necross complication Ventricular arrhythmia HF cardiogenic shock

Libman sacks endocarditis

In SLE non bacterial, verrucous thrombi usually on mitral or aortic valve and can be present on either surface of the valves (usually on undersurface)

Acute pericarditis

Inflammation of the pericardium sharp pain, aggrevated by inspiration, relieved by sitting up and leaning forward pericardial effusion friction rub ECG: widespread ST-segement elevation and /or PR depression causes: idiopathic (most common, presumed viral) confirmed infection (eg, coxsackievirus) neoplasia antoimmune (SLE, rhematoid arthritis) uremia cardiovascular (acute STEMI, or dressler syndrome): focal radiation therapy

Coronary infarction areas affected

LAD infarction: 1) Anterior wall of LV 2) Anterior septum of LV LCX infarction: Lateral wall of the LV RCA infarction: 1) Posterior wall of the heart 2) Posterior septum 3) Papillary muscles of the LV 4) AV and SA nodes LAD > RCA > LCX

murmurs of mitral valve prolapse

Late systolic crescendo murmur with midsystolic click (MC)( due to sudden tensing of chordae tendineae) best heard over apex can predisposed to infective endocarditis can be caused by myxomatous degeneration ( Manfan syndrome or Ehlers- Danlos syndrome), rheumatic fever, chordae rupture

Trousseau syndrome

Migratory thrombophlebitis—redness and tenderness on palpation of extremities

dressler syndrome

Occurs several weeks after MI. Autoimmune phenomenon resulting in fibrinous pericarditis

Mucinous cystadenocarcinoma

Pseudomyxoma peritonei-intraperitoneal accumulation of mucinous material from ovarian or appendiceal tumor.

Holosystolic murmur

Tricuspid area: VSD tricuspid regurgitation Mitral area: mitral regurgitation

persistent truncus arteriosus

Truncus arteriosus fails to divide into pulmonary trunk and aorta due to lack of aorticopulmonary septum formation. most patients have accompanying VSD. 22q11 syndromes

Chagas disease

Trypanosoma cruzi dilated cardiomyopathy with apical atrophy, megacolon, megaesophagus South America Unilateral periorbital swelling characteristic of acut stage reduviid bug faces trypomastigote in blood smear Benznidazole nifurtimox (cruzing in my benz, with a fur coat on)

Coarction of the aorta

aortic narrowing near insertion of ductus arteriosus (juxtaductal) associated with bicuspid aortic valve, other heart defects and Turner syndrome hypertension in upper extremities and weak, delayed pulse in lower extremities (brachial - femoral delay ) with age, intercostal arterie eorde ribs --- notched appearance on CXR. Complication include HF, ^ risk of cerebral hemorrhage (berry aneurysms), aortic rupture, and possible endocarditis.

brugada syndrome

autosomal dominant disorder most often in Asian males ECG: pseudo-right bundle branch block and ST elevation in V1--V3 ^ risk of ventricular tachyarrhythmias and SCD prevent with implantable cardioverter defibrillator (ICD)

angina

cheset pain ischemic myocardium 2° to coronary artery narrowing or spasm no myocyte necrosis stable: 2 ° to atherosclerosis exertional chest pain in classic distribution ( usually with ST depression on ECG ) resolving with rest or nitroglycerin Variant (prinzmetal): at rest 2° to coonary artery spasm transient ST elevation on ECG smoking is a factor, but hypertension and hypercholesterolemia are not triggers may include cocanine, alcohol, and triptans treat with Ca2+ channel blockers, nitrates, and smoking cessation Unstable thrombosis with incomplete coronary artery occlusion +/− ST depression and/or T-wave inversion on ECG no cardiac biomarker elevation (unlike NSTEMI); ^ in frequency or intensity of chest pain or any chest pain at rest.

Sudden cardiac death

death from cardic causes within 1 hour of onset of symptoms most commonly due to a lethal arrhythmia associated with CAD cardiomyopathy (hypertrophic, dilated) hereditary ion channelopathies long Q-T syndrome Brugada syndrome prevent with implantable cardioverter defibrillator (ICD)

Heart failure

dyspnea orthopnea fatigue signs include S3 heart sound rales jugular venous distention pitting edema systolic dysfunction reduced EF ^ EDV(end-diastolic volume) ! contractility often 2 ° to ischemia/MI or dilated cardiomyopathy diastolic dysfunction preserved EF normal EDV ! compliance often 2 ° to myocardial hypertrophy ACEI or angiotensin II blockers, b-blockers (except in acute decompensated HF) and spironolactone ! mortality Thiazide or loop diuretics are used mainly for symptomatic relief. Hydralazine with nitrate therapy improves both symptoms and mortality in select patients

Rheumatic fever

group A b-hemolytic streptococci rheumatic heart disease: mitral> aortic>tricupsid mitral regurgitation -----mitral stenosis Aschoff bodies ( granuloma with giant cells) Anitschkow cells ( enlarged macrophages with ovoid, wavy, rod-like nucleus) Anti streptolysin O titers Immune mediated (type II hypersensitivity) Antibodies of M protein cross-react with self antigens (molecular mimicry) penicillin J♥NES (major criteria): Joint (migratory polyarthritis) ♥ (carditis) Nodules in skin (subcutaneous) Erythema marginatum Sydenham chorea

right heart failure

hepatomegaly (nutmeg liver) ^ central venous pressure---^ resistent to portal flaw rarely leads to cardiac cirrhosis Jugular venous distention peripheral edema COPD

patent ductus arteriosus

in fetal peroid, shunt is right to left (normal) in neonatal peroid, ! pulmonary vascular resistance---shunt becomes left to right---progressive RVH and/or LVH and HF a continuous machine like murmur patency is maintained by PGE synthesis and low O2 tension. PDA can eventually result in late cyanosis in the lower extremities (differential cyanosis) "Endomethacin" (indomethacin) ends patency of PDA; PGE keeps ductus Going (may be necessary to sustain life in conditions such as transposition of the great vessels).

Diagnosis of myocardial infarction

in the first 6 hours, ECG is the gold standard Troponin I 4 hours 24hours peak ^ 7-10days CK-MB 6-12hours 16-24 hours peak 48hours returen to normal

Congenital long QT syndrome

inherited disorder of myocardial repolarization typically due to ion channel defects ^ the risk of sudden cardiac death due to torsades de points includes: Romano-Ward syndrome: autosomal dominant pure cardiac phenotype Jervell and Lange-nielsen syndrome: autosomal recesive sensorineural deafness

Atrial septal defect

loud S1 wide fixed split S2 Ostium secundum defects Ostium primum defects Distinct from patent foramen ovale in that septa are missing tissue rather than unfused may lead to paradoxical emboli systemic venous emboli use ASD to bypass lungs and become systemic arterial emboli

Torsades de points

polymorphic ventricular tachycardia shifting sinusoidal waveforms on ECG progress to ventricular fibrillation (VF) long QT interval predisposes to torsades de points caused by drugs, ! K+, !Mg2+, congenital abnormalities drugs (ABCDE) antiarrythmics (class IA, III) antibiotics (eg, macrolides) anti"c"ychotics (eg, haloperidol) antidepressants (eg, TCAs) antiemetics (eg, ondansetron) treatment includes magenesium sulfate

restrictive/ infiltrative cardiomyopathy

postradiation fibrosis loffler syndrome Endocardial fibroelastosis (think fibroelastic tissue in endocardium of young children) Amyloidosis sarcoidosis hemochromatosis( although dilated cardiomyophaty is more common) (Puppy leash) Diastolic dysfunction ensues. low voltage ECG Loffler syndrome: endomyocardial fibrosis with a prominent eosinophilic infiltrate

Total anomalous pulmonary venous return

pulmonary veins drain into right heart circulation (Superior vena cava, coronary sinus, etc) associated with ASD and sometimes PDA to allow for right-to-left shunting to maintain cardic output.

Eisenmenger syndrome

uncorrected left to right shunt (VSD, ASD, PDA)----^ pulmonary blood flow-----pathologic remodeling of vasculature---pulmonary arterial hypertension. RVH occures to compensate---shunt becomes right to left. causes late cyanosis, clubbing and polycythemia age of onest varies


Kaugnay na mga set ng pag-aaral

Foundations of Western Culture14-27

View Set

Anatomy and Physiology: Chapter 6,7,&8 Review

View Set

William Howard Taft's Domestic Policy

View Set

MH Exam #2 | Chapter 13 PrepU Questions

View Set

ch 4 Macroeconomics: Price Ceilings, floors, binding, and non binding

View Set

Micro 4000: Chapter 16 Innate Immunity

View Set

Chapter 15.8 (1/3) - AZ Life & Health State Laws

View Set